SlideShare a Scribd company logo
1 of 44
Download to read offline
1
1
Giải tích toán học. Tập 1. NXB Đại học quốc gia Hà Nội 2007.
Từ khoá:Giải tích toán học, giải tích, Phép tích vi phân, Đạo hàm, vi phân, Công thức
Taylor, Khai triển Maclaurin, Quy tắc L’hospital.
Tài liệu trong Thư viện điện tử ĐH Khoa học Tự nhiên có thể được sử dụng cho mục
đích học tập và nghiên cứu cá nhân. Nghiêm cấm mọi hình thức sao chép, in ấn phục
vụ các mục đích khác nếu không được sự chấp thuận của nhà xuất bản và tác giả.
Mục lục
Chương 4 Phép tính vi phân của hàm một biến ....................................................................... 2
4.1 Đạo hàm và cách tính ....................................................................................................... 3
4.1.1 Định nghĩa đạo hàm................................................................................................... 3
4.1.2 Công thức đối với số gia của hàm số......................................................................... 3
4.2 Các qui tắc tính đạo hàm .................................................................................................. 4
4.2.1 Các qui tắc tính đạo hàm............................................................................................ 4
4.2.2 Đạo hàm của hàm số hợp........................................................................................... 4
4.2.3 Đạo hàm của hàm số ngược....................................................................................... 6
4.2.4 Đạo hàm theo tham số................................................................................................ 7
4.2.5 Đạo hàm một phía...................................................................................................... 7
4.2.6 Đạo hàm vô cùng ....................................................................................................... 9
4.2.7 Đạo hàm các hàm số sơ cấp....................................................................................... 9
4.3 Vi phân của hàm số ........................................................................................................ 10
4.3.1 Định nghĩa................................................................................................................ 10
Chương 4. Phép tính vi phân của hàm một biến
Lê Văn Trực
2
4.3.2 Các qui tắc tính vi phân ........................................................................................... 11
4.3.3 Vi phân của hàm số hợp........................................................................................... 11
4.3.4 Ứng dụng của vi phân............................................................................................. 12
4.4 Các định lí cơ bản của hàm khả vi.................................................................................. 12
4.8.1 Cực trị địa phương ................................................................................................... 12
4.5 Đạo hàm và vi phân cấp cao........................................................................................... 18
4.8.1 Định nghĩa đạo hàm cấp cao.................................................................................... 18
4.8.2 Các công thức tổng quát đối với đạo hàm cấp n...................................................... 18
4.8.3 Vi phân cấp cao........................................................................................................ 19
4.6 Công thức Taylor............................................................................................................ 20
4.8.1 Công thức Taylor ..................................................................................................... 20
4.8.2 Khai triển Maclaurin................................................................................................ 22
4.7 Qui tắc L’hospital để khử dạng vô định ......................................................................... 25
4.8.1 Dạng vô định
0
0 ....................................................................................................... 25
4.8.2 Dạng vô dịnh
∞
∞ ...................................................................................................... 27
4.8 Khảo sát hàm số.............................................................................................................. 30
4.8.1 Khảo sát đường cong cho dưới dạng phương trình hiện.......................................... 30
4.8.2 Đường cong cho dưới dạng tham số........................................................................ 32
4.8.3 Khảo sát đường cong trong tọa độ cực .................................................................... 36
4.9 Bài tập chương 4............................................................................................................. 39
Chương 4
3
3
Phép tính vi phân của hàm một biến
4.1 Đạo hàm và cách tính
4.1.1 Định nghĩa đạo hàm
Giả sử U là một tập mở trong , :f U → và 0x U∈ .
Cho x0 một số gia 0xΔ ≠ đủ nhỏ sao cho 0x x U+ Δ ∈ . Khi đó ta gọi
0 0( ) ( )y f x x f xΔ = + Δ − là một số gia của hàm số tương ứng với số gia đối số xΔ tại điểm x0.
Xét tỷ số giữa số gia hàm số với số gia đối số.
Nếu tỷ số dẫn đến một giới hạn hữu hạn xác định khi 0xΔ → , thì ta nói rằng hàm f khả vi
tại điểm x0, giới hạn đó gọi là đạo hàm của hàm số tại x0 và ký hiệu là
0 0
0
0
( ) ( )
( ) lim
x
f x x f x
f x
xΔ →
+ Δ −
′ =
Δ
. (4.1.1)
Các ký hiệu y′ hay ( )f x′ là các ký hiệu đạo hàm theo Largrange, còn
dy
dx
hay 0( )df x
dx
là
các kí hiệu theo Leibnitz và Dy hay Df(x0) là các kí hiệu theo Cauchy.
Đôi khi để nhấn mạnh biến số lấy đạo hàm, người ta thường viết biến đó thành chỉ số
dưới:
0 0, ( ), hay ( )′ ′x x x xy f x D y D f x (4.1.2)
Hàm f được gọi là khả vi trên U nếu nó khả vi tại mọi điểm thuộc U.
4.1.2 Công thức đối với số gia của hàm số
Nếu hàm y = f(x) khả vi tại 0 ,∈x U ta có thể biểu diễn số gia của hàm số
0 0 0( ) ( ) ( )Δ = Δ = + Δ −y f x f x x f x như sau.
Theo định nghĩa 0
0
0
( )
lim ( )
Δ →
Δ
′=
Δx
f x
f x
x
.
Đặt 0
0
( )
( ) α
Δ
′= +
Δ
f x
f x
x
với 0α → khi 0Δ →x . (4.1.3)
Ta có 0 0( ) ( ) .α′Δ = Δ + Δf x f x x x với
0
lim 0α
Δ →
→
x
. (4.1.4)
Kí hiệu . ( )α Δ = ο Δx x và hiển nhiên
0
( )
lim 0
Δ →
ο Δ
=
Δx
x
x
.
Do đó (4.1.4) có thể viết dưới dạng
0 0( ) ( ) ( ).′Δ = Δ + ο Δf x f x x x (4.1.5)
Định lý 4.1.1 Nếu hàm y = f(x) khả vi tại 0x U∈ thì f(x) liên tục tại x0.
Chứng minh: Thật vậy ta có
4
0 0 0( ) ( ) ( ) ( )′+ Δ − = Δ + ο Δf x x f x f x x x ,
suy ra
[ ]0 0 0
0 0 0
0 0
0
lim ( ) ( ) lim ( ) lim ( )
lim ( ) ( ).
Δ → Δ → Δ →
Δ →
′+ Δ − = Δ + ο Δ
⇒ + Δ =
x x x
x
f x x f x f x x x
f x x f x
4.2 Các qui tắc tính đạo hàm
4.2.1 Các qui tắc tính đạo hàm
Trước hết ta hãy nhắc lại các qui tắc tính đạo hàm đã biết
Định lí 4.2.1 Cho , :f g U → , trong đó U là tập hợp mở trong R, còn f, g là hai hàm khả vi
tại 0x U∈ . Khi đó 1 2,c c∀ ∈ các hàm 1 2 ,c f c g+ .f g và
f
g
(nếu g(x0) 0≠ cũng là các hàm
khả vi tại điểm x0 và ta có các công thức sau:
a) 1 2 0 1 0 2 0( ) ( ) ( ) ( )c f c g x c f x c g x′ ′ ′+ = + (4.2.1)
b) 0 0 0 0 0( , ) ( ) ( ) ( ) ( ). ( )f g x f x g x g x f x′ ′ ′= + (4.2.2)
c) 0 0 0 0
0 02
0
0
( ) ( ) ( ). ( )
( ) , ( )
( )
f x g x g x f xf
x g x
g g x
′ ′ ′−⎛ ⎞
= ≠⎜ ⎟
⎝ ⎠
. (4.2.3)
4.2.2 Đạo hàm của hàm số hợp
Định lí 4.2.2 Cho :g U V→ và :f V → trong đó U, V là hai tập hợp mở trong , hàm
u=g(x) khả vi tại 0x U∈ và hàm y=f(u) khả vi tại u0=g(x0) V∈ . Khi đó hàm hợp 0f g khả vi tại
x0 và ta có công thức
0 0 0 0( ) ( ) ( ( )) ( )f g x f g x g x′ ′ ′= (4.2.4)
hay gọn hơn
.x u xy y u′ ′ ′= . (4.2.5)
Chứng minh: Theo công thức (4.1.5) hàm f khả vi tại u0, nên ta có
0 0 0( ) ( ) ( ) ( )uf f u u f u f u u u′Δ = + Δ − = Δ + ο Δ .
Mặt khác hàm g khả vi tại x0 nên
0 0 0( ) ( ) ( ) ( )xu g x x g x g x x x′Δ = + Δ − = Δ + ο Δ .
Thế uΔ vào biểu thức fΔ ta được
[ ]0 0 0 0
0 0 0
( ) ( ) ( ) ( ) ( ) ( )
= ( ). ( ) ( ) ( ) ( ).
u x
u x u
f u u f u f u g x x x u
f u g x x f u x u
ο
ο
′ ′+ Δ − = Δ + Δ + ο Δ
′ ′ ′Δ + Δ + ο Δ
Chia cả 2 vế cho xΔ
5
5
0 0
0 0 0
( ) ( ) ( ) ( )
( ). ( ) ( ) .u x u
f u u f u x u
f u g x f u
x x x
+ Δ − ο Δ ο Δ
′ ′ ′= + +
Δ Δ Δ
Ta thấy do hàm u liên tục tại x0 nên khi 0xΔ → thì 0uΔ → và
0 0 0
0
( ) ( ( )) ( ),
( ) ( ) ( ( )) ( ).
o
o
f u f g x f g x
f u u f u f g x f g x
= =
+ Δ = = =
Bây giờ ta hãy viết lại biểu thức trên dưới dạng:
0 0 0
0 0 0
( ) ( ) ( ) ( )
( ). ( ) ( ) . .u x u
f g x f g x x u u
f u g x f u
x x u x
− ο Δ ο Δ Δ
′ ′ ′= + +
Δ Δ Δ Δ
Cho 0xΔ → ta được 0 0 0 0( ) ( ) ( ( )). ( ),uf g x f g x g x′ ′ ′= và công thức được chứng minh.
Ví dụ 3:
i) Ta thấy 0lnx x a
a e a= ∀ >
nên ln
( ) ( )x x a
a e′ ′= , đặt u = xlna, ln
( )' .ln lnu x a x
e e a a a= =
Do đó ta có công thức sau
ln( )x x
a a′ = a với 0a∀ > . (4.2.6)
ii) Ta có 0ln
x e xα α
= ∀ >x
và α∀ ∈
Do đó:
1 1ln ln
( ) ( ) . . . .x x
x e e x
x x
α α α α
α α′ ′= = = .
Và ta có công thức sau:
1
( ) .x xα α
α −
′ = . (4.2.7)
Ví dụ 4: Tính
1
1
cos
x
x
d
I e
dx
−
+
= với 1x ≠ −
Đặt
1
1
cos
x
u
x
−
=
+
1
1
1
1
cos
. . cos
x
u u x
x
d x
I e e u e
dx x
−
+
′−⎛ ⎞
′= = = ⎜ ⎟+⎝ ⎠
Lại đặt
1
1
x
v
x
−
=
+
ta có
2
1 1 1 2
1 1 1 1
(cos ) sin . sin sin .
( )
x x x
v v v
x x x x
′− − −⎛ ⎞
′ ′= − = − = −⎜ ⎟+ + + +⎝ ⎠
Cuối cùng
1
1
2
1 1
2
11
cos
. .sin
( )
x
x
x
I e
xx
−
+
−⎛ ⎞
= − ⎜ ⎟++ ⎝ ⎠
.
Ví dụ 5: Cho , :f g U → trong đó f(x)>0, x U∀ ∈ và tồn tại ( ), ( )f x g x′ ′ với x U∈ .
6
Khi đó
( ) ( )ln ( ) ( )ln ( )
( )
( ( )) ( ( ).ln ( ))
( )
=( ( )) . ( )ln ( ) ( ). .
( )
g x g x f x g x f x
g x
d d d
f x e e g x f x
dx dx dx
f x
f x g x f x g x
f x
⎡ ⎤ ⎡ ⎤= =⎣ ⎦ ⎣ ⎦
′⎡ ⎤
′ +⎢ ⎥
⎣ ⎦
4.2.3 Đạo hàm của hàm số ngược
Định lí 4.2.4 Giả sử hàm f(x) khả vi liên tục trên (a,b) với 0( )f x′ ≠ ( , )x a b∀ ∈ . Khi đó hàm
f(x) đơn điệu thực sự nên có hàm ngược x = g(y), : ( ( ), ( )) ( , ).g f a f b a b→
Khi đó g(y) cũng khả vi tại y = f(x) và có đạo hàm g’(y) thoả mãn hệ thức:
1
( )
( )
g y
f x
′ =
′
(4.2.8)
hay gọn hơn:
1
y
x
x
y
′ =
′
. (4.2.9)
Chứng minh: Do (g.f)(x) = x ( , )x a b∀ ∈
Hay ( ( ))g f x x= ( , )x a b∀ ∈ .
Lấy đạo hàm hai vế đẳng thức trên theo x ta được
1 hay 1( ( )). ( ) ( ). ( )g f x f x g y f x′ ′ ′ ′= =
suy ra
1
( )
( )
g y
f x
′ =
′
( , )x a b∀ ∈ .
Ví dụ 6:
i) Xét hàm số y = arcsinx với −1< x <1 và
2 2
y
π π
− < < .
Ta biết rằng y = arcsinx, tương đương với x = siny, do đó
do
2 2
cos , ,yx y y
π π⎛ ⎞
′ = ∈ −⎜ ⎟
⎝ ⎠
thì 0cos y >
nên 2
1yx x′ = − , suy ra
2
1
1
xy
x
′ =
−
.
Tương tự, tao có các công thức sau:
ii) y = arccosx với −1< x <1,
2
1
1
xy
x
′ = −
−
iii) y = arctgx với x−∞ < < +∞ , 2
1
1
xy
x
′ =
+
7
7
iv) y = arccotgx với x−∞ < < +∞ , 2
1
1
xy
x
−
′ =
+
.
4.2.4 Đạo hàm theo tham số
Xét hàm y của biến x được cho dưới dạng tham số
( )
( )
x x t
y y t
=⎧
⎨
=⎩
với ( , ).t α β∈
Giả sử x là hàm khả vi, liên tục và '( ) 0x t ≠ ( , )t α β∈ .
Khi đó x(t) là hàm đơn điệu thực sự trên ( , )α β , vì vậy nó có hàm ngược t = t(x). Khi đó
ta có hàm hợp y = y(t) = y(t(x)).
Hãy tính xy′ . Cho t một số gia Δt, Δx là số gia tương ứng của Δt, Δy là số gia tương
ứng của Δx. Ta có
y
y t
xx
t
Δ
Δ Δ=
ΔΔ
Δ
suy ra
0
0
0
lim
lim
lim
t t
x
x
t
t
y
yy ty
xx x
t
Δ →
Δ →
Δ →
Δ
′Δ Δ′ = = =
Δ ′Δ
Δ
. (4.2.10)
Ví dụ 7: Xét hàm số
1( sin ), ( cos )x a t t y a t= − = − với 0 2( , )t π∈ .
Khi đó
2
2
2 2
1 22
2
sin cos
sin
( ) cot g
( cos ) sin
t t
a t t
y x
ta t
′ = = =
−
.
4.2.5 Đạo hàm một phía
Giả sử f(x) được xác định trên (a,b) và 0 ( , )x a b∈ . Ta nói giới hạn hữu hạn, nếu tồn tại
0 0
0 0
( ) ( )
lim lim
x x
f x x f xy
x x+ +
Δ → Δ →
+ Δ −Δ
=
Δ Δ
(4.2.11)
là đạo hàm bên phải của hàm f(x) tại điểm x0, kí hiệu là 0( )+
′f x (xem hình 4.2.1).
Tương tự, ta có đạo hàm bên trái của hàm f(x) tại điểm x0 kí hiệu là 0( ):−
′f x
0 0
0
0 0
( ) ( )
lim lim ( )
x x
f x x f xy
f x
x x− − −
Δ → Δ →
+ Δ −Δ
′= =
Δ Δ
(4.2.12)
Ta thấy muốn có 0( )f x A′ = điều kiện cần và đủ là
0 0( ) ( )f x f x A+ −
′ ′= = .
8
Hình 4.2.1
Ví dụ 8: Cho hàm f(x) =|x|, hãy xét đạo hàm của hàm số tại x0 = 0.
Ta có 0 0y f x f xΔ = + Δ − = Δ( ) ( ) | | ,
0 0
0 0
0 1
0 1
( ) lim lim ,
( ) lim lim .
x x
x x
y x
f
x x
y x
f
x x
+ +
− −
+
Δ → Δ →
−
Δ → Δ →
Δ Δ
′ = = =
Δ Δ
Δ −Δ
′ = = = −
Δ Δ
Vậy hàm f(x) liên tục tại x0 = 0, nhưng f’(0) không tồn tại.
Ví dụ 9: Cho hàm số
3
khi 0
khi 0
sin
( )
x
x
f x x
a x
⎧
≠⎪
= ⎨
⎪ =⎩
1) Tìm a để hàm số liên tục tại x = 0.
2) Với a tìm được, hãy xét sự khả vi của hàm số tại x = 0
Giải: 1) Do
3
2
0 0
0
sin sin
lim lim sin
x x
x x
x
x x→ →
= =
Vậy để hàm liên tục tại x = 0 thì phải có a = 0.
2) Với a=0 ta có
3
khi 0
khi 0
sin
( )
0
x
x
f x x
x
⎧
≠⎪
= ⎨
⎪ =⎩
Ta thấy
3
0 0
0
0
0
( ) ( ) sin
lim lim
x x
f x f x
x x→ →
−
= =
−
.
Vậy 0 0( )f ′ = và hàm khả vi tại x=0.
Ví dụ 10: Chứng minh rằng hàm số f(x) =|x−a| ( )ϕ x , trong đó ( )ϕ x là hàm liên tục và
0( )aϕ ≠ , không khả vi tại x = a.
9
9
Ta có
0 0
( ) ( ) | | ( )
( ) lim lim
x x
f a x f a x a x
f x
x x
ϕ
→ →
+ Δ − Δ + Δ
′ = =
Δ Δ
.
Suy ra:
( )+
′f a = ( )ϕ a và ( )−
′f a =– ( )ϕ a .
Do ( ) ( )+ −
′ ′≠f a f a nên hàm số f(x) không khả vi tại x=a.
4.2.6 Đạo hàm vô cùng
Nếu 0 0
0 0
hay
( ) ( )
lim lim
x x
f x x f xy
x xΔ → Δ →
+ Δ −Δ
= = +∞ − ∞
Δ Δ
thì ta nói rằng tại x = x0 hàm f(x) có
đạo hàm vô cùng. Khi đó tiếp tuyến với đồ thị f(x) tại x = x0 song song với trục Oy.
Ta cần chú ý rằng nếu như 0( )f x′ không là hữu hạn thì hàm f(x) không nhất thiết phải
liên tục tại điểm x0. Ví dụ xét hàm
1 khi 0
0 khi 0
1 khi 0
( )
.
x
f x x
x
− <⎧
⎪
= =⎨
⎪ >⎩
Với 0xΔ ≠ , ta có
0 1( ) ( )
| |
f x f
x x
Δ −
=
Δ Δ
, do đó 0( )f ′ = +∞ nhưng đương nhiên f(x) không
liên tục tại điểm x0 = 0.
4.2.7 Đạo hàm các hàm số sơ cấp
Sau đây là bảng đạo hàm của một số hàm sơ cấp:
1 0
2 1
)
)
y c y
y x y
′= =
′= =
1
2
3 1
1 1
1
2
) , , .y x R y x
y y
x x
y x y
x
α α
α α α −
′= ∈ ≠ − =
−
′= =
′= =
4) x x
y e y e′= =
x
y a= với 0 lnx
a y a a′> =
5) logay x= với
1
0
ln
a y
x a
′> =
1
lny x y
x
′= =
6) sin cosy x y x′= =
10
7) cos siny x y x′= = −
2
2
1
8 tg) sec
cos
y x y x
x
′= = =
2
2
1
9) cot g cosec
sin
y x y x
x
′= = − = −
2
1
10) arcsin
1
y x y
x
′= =
−
2
1
11) arccos
1
y x y
x
′= = −
−
2
1
12) arctg
1
y x y
x
′= =
+
2
1
13) arccot g
1
y x y
x
′= = −
+
14) sh chy x y x′= =
15) ch shy x y x′= =
2
1
16) th
ch
y x y
x
′= =
2
1
17) cth
sh
y x y
x
−
′= =
2
1
18) argsh
1
y x y
x
′= =
+
2
1
19) arg ch
1
y x y
x
′= =
−
2
1
20) arg th
1
y x y
x
′= =
−
2
1
21) arg cth .
1
y x y
x
′= =
−
4.3 Vi phân của hàm số
4.3.1 Định nghĩa
Cho hàm y = f(x) xác định trên tập hợp mở U ⊂ và 0x U∈ . Cho x0 một số gia 0xΔ ≠
đủ nhỏ sao cho 0x x U+ Δ ∈ .
Giả sử f(x)khả vi tại 0x U∈ , khi đó
0 0 0( ) ( ) ( ) ( )f x x f x f x x xο′+ Δ − = Δ + Δ . (4.3.1)
11
11
Ta gọi biểu thức 0( )f x x′ Δ là vi phân của hàm f(x) tại điểm x0 ứng với số gia xΔ của đối
số và kí hiệu là
0 0( , ) ( )df x x f x x′Δ = Δ . (4.3.2)
Bây giờ ta xét trường hợp đặc biệt khi f(x) = x. Ta có ( ) 1f x′ = , do đó dx = 1. xΔ = xΔ , vì
thế trong biểu thức (4.3.2) ta có thể viết dx thay cho xΔ và dx gọi là vi phân của biến số độc
lập. Từ đây, ta có thể xác định vi phân của hàm f tại x U∈ theo công thức
Df = ( )f x′ dx (4.3.3)
hay dy = ( )y x′ dx (4.3.4)
Hệ thức này giải thích lí do ta kí hiệu đạo hàm của hàm y = f(x) là ( )
dy
y x
dx
′ = .
4.3.2 Các qui tắc tính vi phân
Từ các qui tắc tính đạo hàm, ta dễ dàng suy ra các qui tắc tương ứng cho vi phân.
1 2 1 2 1 2) ( ) ,i d c f c g c df c dg c c+ = + ∀ ∈
) ( . )ii d f g gdf fdg= + (4.3.5)
2
) ( ) nÕu 0
f gdf fdg
iii d g
g g
−
= ≠ .
4.3.3 Vi phân của hàm số hợp
Giả sử các hàm y = f(x) và x = g(t) sao cho đối với chúng có thể thiết lập hàm hợp y =
f(g(t)). Nếu tồn tại các đạo hàm xy′ và tx′ thì theo quy tắc đạo hàm hàm hợp sẽ tồn tại đạo
hàm
ty′ = xy′ . tx′ . (4.3.6)
Nếu xem x là biến độc lập thì vi phân dy được biểu thị bởi công thức (4.3.4). Bây giờ ta
xem x là hàm của biến t, ta có
tdy y dt′= (4.3.7)
Tuy nhiên nếu thay đạo hàm ty′ bởi biểu thức (4.3.6) và chú ý rằng dx = tx′dt, thì cuối
cùng ta được
x t xdy y x dt y dx′ ′ ′= =
hay dy = ( )y x′ dx, tức là quay trở lại dạng ban đầu của vi phân.
Như vậy, ta luôn luôn có quyền viết vi phân của y dưới dạng (4.3.4) dù x có phải là biến
độc lập hay không . Điều khác nhau chỉ là ở chỗ, nếu chọn t là biến độc lập thì dx không phải
là số gia tuỳ ý mà là vi phân của x xem là hàm của t. Tính chất đó gọi là tính bất biến của
dạng vi phân,
Ví dụ 1: Cho hàm số
1
ln
1
x
x
e
y
e
+
=
−
, hãy tính dy
12
Ta thấy
2 2
1 1 2 2
.
1 1 1 1
x x x x
x x x x
e e e e
y dy dx
e e e e
′⎛ ⎞− + −
′ = = ⇒ = −⎜ ⎟
+ − − −⎝ ⎠
Ví dụ 2: Tính:
(sin )
(cos )
d x
d x
Ta có:
(sin ) cos
cot g
(cos ) sin
d x xdx
x
d x xdx
= = −
−
với , .x k kπ≠ ∈
4.3.4 Ứng dụng của vi phân
Cho hàm y = f(x) xác định trên tập mở U ⊂ và 0x U∈ . Giả sử f khả vi tại 0x U∈ . Cho
x0 một số gia h sao cho 0x h U+ ∈ , khi đó
0 0 0 0( , ) ( ) ( ) ( ) ( )f x h f x h f x f x h hο′Δ = + − = + . (4.3.8)
Nếu |h| đủ nhở thì ( )hο nhỏ tuỳ ý và ta có xấp xỉ
0 0 0( ) ( ) ( )f x h f x f x h′+ − ≈
hay
0 0 0( ) ( ) ( )f x h f x f x h′+ ≈ + . (4.3.9)
Ví dụ 3: Tính gần đúng arctg1,05 .
Theo công thức (4.3.9), ta có
12
1
arctg1,05 arctg1 | .0,05 0,81
1
x
x
=≈ + ≈
+
.
Ví dụ 4: Tính gần đúng arcsin 0,05
Theo công thức (4.3.9), ta có
0
2
1
arcsin 0,05 arcsin 0 | .0,05 0.05
1
x
x
=≈ + =
−
.
4.4 Các định lí cơ bản của hàm khả vi
4.8.1 Cực trị địa phương
Cho hàm f(x) xác định trên khoảng (a,b). Ta nói rằng hàm f(x) đạt cực đại địa phương tại
điểm ( , )c a b∈ nếu tồn tại một số 0δ > sao cho
( ) ( ) ( , ).f x f c x c cδ δ≤ ∀ ∈ − + (4.4.1)
Hàm f đạt cực tiểu địa phương tại ( , )c a b∈ nếu:
( ) ( ) ( , )f x f c x c cδ δ≥ ∀ ∈ − + . (4.4.2)
Điểm mà tại đó hàm đạt cực đại hoặc cực tiểu địa phương gọi chung là điểm cực trị.
13
13
Định lí Ferma Cho : ( , )f a b → , nếu hàm đạt cực trị tại ( , )c a b∈ và nếu f(x) khả vi tại c thì
( ) 0f c′ = . (4.4.3)
Chứng minh:
Giả sử hàm đạt cực đại tại c (trường hợp đạt cực tiểu tại c chứng minh tương tự).
Do hàm đạt cực đại tại c nên ∀h đủ nhỏ ta có
0( ) ( )f c h f c h+ − ≤ ∀
suy ra 0 0
( ) ( )f c h f c
h
h
+ −
≤ ∀ >
0 0
( ) ( )f c h f c
h
h
+ −
≥ ∀ < .
Cho nên
0
0
( ) ( )
( ) lim
h
f c h f c
f c
h++
→
+ −
′ = ≤ và
0
0
( ) ( )
( ) lim
h
f c h f c
f c
h−−
→
+ −
′ = ≥ .
Mặt khác vì f có đạo hàm tại điểm c nên ( ) ( ) ( )f c f c f c+ −
′ ′ ′= = , do đó 0( )f c′ = (xem hình
4.4.1)
Hình 4.4.1
Chú ý rằng sự triệt tiêu của đạo hàm ( )′f c về phương diện hình học có ý nghĩa là tiếp
tuyến tại điểm tương ứng của đường cong song song với trục Ox.
Định lý Rolle Cho hàm :[ , ]f a b → có tính chất sau:
i) f(x) liên tục trên [a,b],
ii) f(x) khả vi trên (a,b),
iii) f(a)=f(b).
Khi đó tồn tại ít nhất một điểm ( , )c a b∈ sao cho ( ) 0f c′ = .
Chứng minh:
Do f(x) liên tục trên đoạn [a,b] nên theo định lí Weierstrass thứ hai hàm f(x) sẽ đạt giá trị
lớn nhất M và giá trị bé nhất m trên đoạn [a,b]:
[ , ] [ , ]
max ( ), min ( ).
x a b x a b
M f x m f x
∈ ∈
= =
Ta hãy xét hai khả năng có thể xảy ra:
14
1) M = m. Khi đó từ bất đẳng thức
( ) [ , ]m f x M x a b≤ ≤ ∀ ∈ suy ra ( ) , [ , ]f x m x a b= ∀ ∈
Vì vậy 0( ) , [ , ]f x x a b′ = ∀ ∈ . Do đó điểm c là lấy điểm bất kì thuộc khoảng (a,b).
2) m<M. Do f(a)=f(b), hàm f(x) không thể đạt cả hai giá trị m, M tại hai đầu mút của
khoảng, có nghĩa là ít nhất một trong hai giá trị đó đạt tại một điểm ( , )c a b∈ . Khi đó, theo
định lí Fermat ( ) 0f c′ = . Định lí đã được chứng minh.
Chú ý:
Ta chú ý rằng giả thiết f(x) liên tục trên [a,b] là một giả thiết không thể bỏ qua được. Ví
dụ như xét hàm số (xem hình 4.4.2)
khi 0 1
1 khi 0
( )
x x
f x
x
< ≤⎧
= ⎨
=⎩
Cho dù f(0)=f(1), nhưng hàm số không liên tục trên [0,1], nên không thể áp dụng định lí
Rolle được (đạo hàm không nơi nào bằng 0 trên (0,1)).
Giả thiết hàm f(x) khả vi trong khoảng (a,b) cũng là một giả thiết không thể bỏ qua được.
Chẳng hạn xét hàm số (xem hình 4.4.3)
1
2
1
2
Hình 4.4.2 Hình 4.4.3
1
ví i 0
2( )
1
1 ví i 1
2
x x
f x
x x
⎧
≤ ≤⎪⎪
= ⎨
⎪ − < ≤
⎪⎩
Hàm số này liên tục trên đoạn [0,1], f(0)=f(1), nhưng không có đạo hàm tại
1
2
x = , do đó
cũng không áp dụng định lí Rolle được.
Ví dụ 1: Hàm số f(x)=1− 23
x triệt tiêu khi x1=−1, x2=1 nhưng ′ ≠( ) 0f x với |x|≤ 1 . Điều
này không mâu thuẫn với định lí Rolle.
Định lí về số gia hữu hạn (Định lí Lagrange).
Giả sử :[ , ]f a b → có các tính chất
i) f liên tục trên [a,b]
15
15
ii) f khả vi trên (a,b)
Khi đó tồn tại ít nhất một điểm ( , )c a b∈ sao cho:
( ) ( )
( )
f b f a
f c
b a
−
′=
−
. (4.4.4)
Chứng minh: Ta hãy xét hàm bổ trợ sau:
( ) ( )
( ) ( ) ( ) ( )
f b f a
F x f x f a x a
b a
−
= − − −
−
.
Hiển nhiên F(x) liên tục trên [a,b] vì nó là hiệu của hàm liên tục f(x) và hàm tuyến tính.
Trong khoảng (a,b) hàm đó có đạo hàm hữu hạn bằng:
( ) ( )
( ) ( ) .
f b f a
F x f x
b a
−
′ ′= −
−
y
xbca0
C
B
A
f(b)
f(a)
Hình 4.4.4
Cuối cùng ta thấy F(a)=F(b)=0. Theo định lí Rolle tồn tại một điểm ( , )c a b∈ sao cho
( ) 0F c′ = . Như vậy
( ) ( )
( ) 0
f b f a
f c
b a
−
′ − =
−
.
Do đó
( ) ( )
( )
f b f a
f c
b a
−
′ =
−
Ý nghĩa hình học:
Tỷ số
( ) ( )f b f a
b a
−
−
là hệ số góc của cát tuyến AB, còn ( )f c′ là hệ số góc của tiếp tuyến
với đường cong y=f(x) tại điểm C(c,f(c)).
Theo định lí Lagrange trên cung AB tìm được ít nhất một điểm c, mà tại đó tiếp tuyến
song song với dây cung AB. Trường hợp f(a) = f(b) ta có định lí Rolle.
Chú ý 1: Bởi vì ( , )c a b∈ , nên ta có thể viết
c = a + ( ), 0 1b aθ θ− < < .
Khi đó công thức Lagrange có thể viết dưới dạng
16
f(b) − f(a )= [ ( )]( ),0 1f a b a b aθ θ+ − − < < . (4.4.5)
Chú ý 2: Nếu đặt a = x, b = x+ xΔ thì ta nhận được
f(x + Δx) − f(x)=f’(x + Δθ x)Δx trong đó θ< <0 1 (4.4.6)
Định lí Cauchy
Giả thiết
i) Các hàm f(x) và g(x) xác định và liên tục trên[a,b]
ii) f(x) và g(x) khả vi trên (a,b)
iii) ′ ≠ ∀ ∈( ) 0 ( , ).g x x a b
Khi đó tồn tại ít nhất một điểm ( , )c a b∈ sao cho
( ) ( ) ( )
( ) ( ) ( )
f b f a f c
g b g a g c
′−
=
′−
. (4.4.7)
Rõ ràng rằng định lí Lagrange là trường hợp đặc biệt của định lí Cauchy: Để được công
thức số gia hữu hạn thì trong công thức Cauchy (4.4.5) ta đặt g(x)=x.
Chứng minh:
Trước hết ta để ý rằng theo định lí Lagrange ta có thể tìm được một số 1 ( , )c a b∈ sao cho:
1( ) ( ) ( )( )g b g a g c b a′− = −
Theo giả thiết 1( ) 0g c′ ≠ , nên ( ) ( ) 0.g b g α− ≠ (4.4.8)
Bây giờ ta xét hàm số
( ) ( )
( ) ( ) ( ) ( ( ) ( ))
( ) ( )
f b f a
F x f x f a g x g a
g b g a
−
= − − −
−
. (4.4.9)
Ta thấy hàm số thoả mãn tất cả các giả thiết của định lí Rolle. Thật vậy F(x) liên tục, đạo
hàm ( )F x′ tồn tại trong khoảng (a,b), cụ thể bằng
( ) ( )
( ) '( ) ( )
( ) ( )
f b f a
F x f x g x
g b g a
−
′ ′= −
−
,
và hiển nhiên F(a)=F(b). Do đó theo định lí Rolle ( , )c a b∃ ∈ sao cho ( ) 0.F c′ =
Nói cách khác
( ) ( )
( ) ( ) 0
( ) ( )
f b f a
f c g c
g b g a
−
′ ′− =
−
.
Từ đây suy ra định lí được chứng minh.
Ví dụ 1: Cho hàm số
2
3
khi 0 1
2( )
1
khi 1 .
x
x
f x
x
x
⎧⎪ −⎪ ≤ ≤⎪⎪⎪= ⎨
⎪⎪ < < +∞⎪⎪⎪⎩
Xác định giá trị trung gian c của công thức số gia hữu hạn đối với hàm số ( )f x trên đoạn
[0,2].
Trước hết ta thấy
17
17
1 1
1
1
( ) (1)
(1) lim lim 1
1 1x x
f x f xf
x x+ ++
→ →
−
−
′ = = = −
− −
2
1 1
3
1
( ) (1) 2(1) lim lim 1
1 1x x
x
f x f
f
x x− −−
→ →
−
−
−
′ = = = −
− −
.
Vậy theo định nghĩa hàm số f(x) có đạo hàm tại x=1 và (1) 1f ′ = − . Do đó ta có
2
2
khi 0 1 khi 0 1
( ) 1 khi 1 1
khi 1< 2
1
khi 1 2
x x x x
f x x
x
x
x
x
⎧
⎪− ≤ < − ≤ ≤⎧
⎪ ⎪
= − = = −⎨ ⎨
≤⎪ ⎪− ⎩⎪ < ≤
⎩
Công thức số gia hữu hạn đối với hàm số f(x) trên [0,2] là
(2) (0) ( )(2 0)f f f c′− = − hay
1 3
2. ( )
2 2
f c′− = ,
suy ra:
1
( )
2
f c
−
′ = .
Mặt khác theo biểu thức đạo hàm ( )f x′ , ta có
Khi 0< c <1,
1
2
c− = − suy ra
1
2
c = ,
Khi 1< c <2, 2
1 1
,
2c
− = − suy ra 2c= .
Ví dụ 2: Giả sử f(x) khả vi trên đoạn [0,1] và (0). (1) 0f f′ ′ < . Chứng minh (0,1)ξ∃ ∈ sao cho
( ) 0f ξ′ = .
Chứng minh: Thật vậy, theo giả thiết hàm số liên tục trên [0,1], nên đạt giá trị lớn nhất,
bé nhất trên [0,1].
Không mất tổng quát, giả sử (0) 0, (1) 0f f+ −
′ ′< > , ta có
0
( ) (0)
(0) lim 0,
x
f x f
f
x++
→
−
′ = <
suy ra
( ) (0)
0 hay ( ) (0) 0
f x f
f x f
x
−
< − < với 0x > khá bé.
Hơn nữa do
1
( ) (1)
(1) lim 0
1x
f x f
f
x−−
→
−
′ = >
−
nên
( ) (1)
0
1
f x f
x
−
>
−
hay ( ) (1)f x f− <0, hay ( ) (1)f x f< khi x khá gần 1, x<1.
Từ lý luận trên suy ra giá trị bé nhất của hàm số trên [0,1] không thể xảy ra ở hai đầu mút
0 và 1. Vậy giá trị bé nhất đạt được tại (0,1)ξ ∈ . Theo định lý Ferma
18
( ) 0f ξ′ =
Ví dụ 3: Chứng minh nếu ( )xϕ là hàm khả vi, đơn điệu tăng và | ( )| ( )f x xϕ′ ′≤ khi 0x x≥ , thì
ta có
0 0 0| ( ) ( )| ( ) ( ) khif x f x x x x xϕ ϕ− ≤ − ≥ .
Chứng minh:
Theo định lý Cauchy
0
0
( ) ( ) ( )
1
( ) ( ) ( )
f x f x f c
x x cϕ ϕ ϕ
′−
= <
′−
với 0x c x< < .
Từ đây suy ra: 0 0 0| ( ) ( )| ( ) ( ) khif x f x x x x xϕ ϕ− ≤ − > . Cuối cùng chú ý rằng đẳng thức
trên hiển nhiên đúng khi x = x0.
4.5 Đạo hàm và vi phân cấp cao
4.8.1 Định nghĩa đạo hàm cấp cao
Giả sử :f U → là hàm khả vi trên tập mở U ⊂ , khi đó ta nhận được hàm
:f U′ → . Nếu tại 0x U∈ , ( )f x′ có đạo hàm thì ta gọi đạo hàm của ( )f x′ tại 0x là đạo
hàm cấp hai của hàm f(x) tại 0x và kí hiệu là 0( )f x′′ . Hàm f có đạo hàm cấp hai tại 0x còn gọi
là khả vi cấp hai tại 0x .
Một cách tổng quát, đạo hàm của đạo hàm cấp (n−1) được gọi là đạo hàm cấp n của hàm
f(x) và kí hiệu là
( ) ( )( )
( ); ; ;
n n
n n
n n
d f x d y
f x y
dx dx
.
Đương nhiên là ( ) ( ) ( )
( ( )) ( )m n m n
f x f x+
= . (4.5.1)
Đôi khi ta viết f(0)
thay cho f.
Ta chú ý rằng, nếu tồn tại ( )
0( )n
f x , tức là nếu hàm ( 1)
( )n
f x−
có đạo hàm tại điểm 0x , thì
hàm ( 1)
( )n
f x−
được xác định không chỉ tại 0x , mà là trong toàn bộ khoảng 0 0( , )x xδ δ− + ,
trong đó là δ số dương được chọn thích hợp. Trong khoảng này những hàm
( 2) ( 3)
( ); ( ),...,n n
f x f x− −
( ), ( )f x f x′ được xác định.
4.8.2 Các công thức tổng quát đối với đạo hàm cấp n
Giả sử :f U → và :g U → là hai hàm khả vi cấp n trên U. Khi đó 1 2 , .c f c f f g+ là
những hàm khả vi cấp n trên U, trong đó 1 2,c c ∈ và
( ) ( ) ( )
1 2 1 2) ( ) ( ) ( ) ( )n n n
i c f c g x c f x c g x+ = + (4.5.2)
( ) ( ) ( )
0
)( . ) ( ) ( ). ( )
n
n k k n k
n
k
ii f g x C f x g x−
=
= ∑ . (4.5.3)
19
19
Công thức (4.5.3) còn gọi là công thức Leibnitz.
Ví dụ 1: Tính đạo hàm cấp 3 của hàm số 3
siny x x=
Đặt 3
, sinf x g x= = . Khi đó
(3) (3) (2) (2) (3)
. 3 . 3 . .y f g f g f g f g′ ′= + + +
hay
(3) 3 2
sin 9 sin 18 cos 6sin .y x x x x x x x= − − + +
Ví dụ 2: Tính đạo hàm cấp 3 của hàm số sau
( ) 3 ( )y xf x a f a x′= − + − trong đó a là hằng số.
Đặt 3
, ( ), ( ), ( )u x v f x a v f a x v f a x′ ′ ′′ ′′= = − = − − = − ,
(3) (4)
( )v f a x= − − ,
khi đó
(3) (3) (4) (3)
(3) (4)
3 ( ) ( ) 3 ( ) hay
( ).
y f a x xf a x f a x
y xf a x
= − − − − −
= − −
4.8.3 Vi phân cấp cao
Cho U mở trong và f là hàm khả vi cấp n trên tập mở U. Ta gọi vi phân cấp hai của
hàm f, ký hiệu là d2
f là biểu thức d2
f=d(df). Một cách tổng quát, ta gọi vi phân cấp n của hàm f
là vi phân của vi phân cấp n−1 của hàm f:
1
( ).n n
d f d d f−
= (4.5.4)
Khi tính vi phân cấp cao ta chú ý rằng dx là một số tuỳ ý và không phụ thuộc x ( dx x= Δ ),
nên khi lấy vi phân theo x phải xem nó là hằng số. Trong trường hợp đó ta sẽ có
2 2
( ) ( ) ( )d y d dy d y dx dy dx y dx dx y dx′ ′ ′′ ′′= = = = = .
3 2 2 3
( ) ( )d y d d y d y dx y dx′′ ′′′= = = .
Bằng cách quy nạp ta chứng minh được rằng
( )n n n
d y y dx= .
Do đó
( )
n
n
n
d y
y
dx
= . (4.5.5)
Như vậy, ký hiệu trên có thể xem như một phân số. Nhờ công thức (4.5.5) ta dễ dàng biến
đổi công thức Leibnitz thành công thức của vi phân. Nhân cả hai vế của (4.5.3) với dxn
ta sẽ
được
0
( ) .
n
n k n k k
n
k
d fg C d f d g−
=
= ∑ (4.5.6)
20
Chú ý trong công thức (4.5.6) ta sẽ xem 0 0
,d f f d g g= = .
Ví dụ 3: Cho y=f(x2
) với f là hàm khả vi. Tính d2
y.
Ta có: 2
2 ( )dy f x xdx′= ,
Lấy vi phân lần thứ hai ta được
2 2 2 2
2 2
2 2 2 2 2
2 ( ) 2 ( ) ( ( ))
2 ( ) ( ).2
2 2 ( ) ( ) .
d y d f x x dx f x dx xd f x dx
f x dx xf x xdx dx
d y x f x f x dx
⎡ ⎤ ⎡ ⎤′ ′ ′= = +⎣ ⎦ ⎣ ⎦
⎡ ⎤′ ′′= +⎣ ⎦
⎡ ⎤′′ ′= +⎣ ⎦
Ví dụ 4: Xét hàm y = arctgx. Ta hãy tính y(n)
theo y.
Vì x = tgy nên 2
2
1
cos cos sin( )
21
y y y y
x
π
′ = = = +
+
Lấy đạo hàm lần thứ hai theo x (và nhớ rằng y là hàm của x) ta được
sin .sin( ) cos .cos( ) .
2 2
y y y y y y
π π⎡ ⎤
′′ ′= − + + +⎢ ⎥
⎣ ⎦
2 2
cos .cos(2 ) cos .sin(2 )
2 2 2
cos .sin 2( ).
2
y y y y
y y
π π π
π
= + = + +
= +
Lấy đạo hàm lần nữa ta được
(3) 2
3 3
2sin cos .sin 2( ) 2cos cos2( ) .
2 2
2cos cos(3 2. ) 2cos sin 3( )
2 2
y y y y y y y
y y y y
π π
π π
⎡ ⎤
′= − + + +⎢ ⎥
⎣ ⎦
= + = +
Một cách tổng quát
( )
( 1)!cos .sin ( )
2
n n
y n y n y
π
= − + .
4.6 Công thức Taylor
Trước đây, ta đã biết nếu hàm f(x) khả vi tại điểm 0x U∈ , trong đó U là tập mở trong ,
thì ta có thể biểu diễn số gia của hàm số dưới dạng
0 0 0( ) ( ) ( ) ( )f x h f x f x h hο′+ − = +
trong đó ( )hο là vô cùng bé bậc cao hơn so với h.
Công thức này cho biết cách tính giá trị của f(x) trong lân cận của điểm x0 khi biết giá trị
f(x0) và đạo hàm 0( )f x′ . Vấn đề đặt ra là nếu biết thêm các đạo hàm cấp cao của hàm f(x) tại
x0, ta có thể biết chính xác hơn giá trị của hàm f(x) trong lân cận x0 hay không?
4.8.1 Công thức Taylor
21
21
Định lí 4.6.1 Cho [ ]: ,f a b → . Nếu hàm f(x) khả vi (n+1) lần trong khoảng (a,b), thì với bất
kì điểm ( , ), ( , )c a b x a b∈ ∀ ∈ mà x c≠ ta luôn có
2
( ) ( 1)
1
( ) ( )
( ) ( ) ( ) ( ) ...
1! 2!
( ) ( )
( ) ( )
! ( 1)!
n n
n n
f c f c
f x f c x c x c
f c f c
x c x c
n n
+
+
′ ′′
= + − + − + +
+ − + −
+
(4.6.1)
trong đó c là một số nằm giữa x và c.
Chứng minh: Trước hết ta hãy tìm đa thức ( )nP x sao cho
( ) ( )
( ) ( ); ( ) ( );...; ( ) ( )n n
n n nP c f c P c f c P c f c′ ′= = = . (6.4.2)
Thật vậy đa thức ( )nP x phải tìm được viết dưới dạng
2
0 1 2( ) ( ) ( ) ... ( )n
n nP x a a x c a x c a x c= + − + − + + − (4.6.3)
khi đó 0( )nP c a= và
1
1 2( ) 1. 2 ( ) ... . ( )n
n nP x a a x c n a x c −′ = + − + + −
1
2
2 3
2
( 1)
1
( )
( ) 2.1. 3.2. ( ) ... ( 1) ( )
( ) 2!
.................
( ) ( 1)( 2)...2.1. ( 1)...3.2.1. ( )
n
n
n n
n
n
n n n
P c a
P x a a x c n n a x c
P c a
P x n n a n n a x c
−
−
−
′ =
′′ = + − + + − −
′′ =
= − − + − −
( )
( ) !n
n nP c n a=
Thay các hệ số a0, a1, …, an vào (4.6.3), đa thức P(x) phải tìm có dạng
( )
2( ) ( ) ( )
( ) ( ) ( ) ( ) ... ( )
1! 2! !
n
n
n
f c f c f c
P x f c x c x c x c
n
′ ′′
= + − + − + + −
(4.6.4)
Bây giờ ta đặt ( ) ( ) ( )n nR x f x P x= − (4.6.5)
Theo giả thiết ( )
( ) ( ) ( ) ... ( ) 0n
n n n nR c R c R c R c′ ′′= = = = =
(4.6.6)
Mặt khác, nếu đặt 1
( ) ( )n
G x x c +
= − (4.6.7)
thì cũng có ( )
( ) ( ) ( ) ... ( ) 0n
G c G c G c G c′ ′′= = = = = và
( 1)
( ) ( 1)!n
G x n+
= +
Giả sử ( , ), ,x a b x c∈ ≠ từ (4.6.6) và (4.6.7) ta có
( ) ( ) ( )
( ) ( ) ( )
n n nR x R x R c
G x G x G c
−
=
−
22
Áp dụng định lí Cauchy vào tỉ số trên ta được
1
1
( ) ( )
( ) ( )
n nR x R c
G x G c
′
=
′
với c1 nằm giữa x và c
Cũng từ các hệ thực (4.6.6) và (4.6.7) ta có
1 1 2
1 1 2
( ) ( ) ( ) ( )
( ) ( ) ( ) ( )
n n n nR c R c R c R c
G c G c G c G c
′ ′ ′ ′′−
= =
′ ′ ′ ′′−
với c2 nằm giữa c1 và c
Như vậy, áp dụng (n+1) lần định lí Cauchy ta được
( 1)
( 1)
( ) ( )
( ) ( )
n
n n
n
R x R c
G x G c
+
+
=
Theo định nghĩa của hàm G(x) ta có
1
( ) ( 1)!,n
G x n+
= + do đó 1
( ) ( 1).n
G c n+
= +
Từ đây suy ra
( 1)
1( )
( ) ( )
( 1)!
n
nn
n
R c
R x x c
n
+
+
= −
+
. (4.6.9)
Từ (4.6.5)
1 1 ( 1) ( 1)
( ) ( ) ( ) ( ).n n n n
n nR x f x P x f x+ + + +
= − =
Từ đây ta nhận được
( 1)
( 1)( )
( ) ( )
( 1)!
n
n
n
f c
R x x c
n
+
+
= −
+
. (4.6.10)
Cuối cùng
2
( ) ( 1)
1
( ) ( ) ( )
( ) ( )
( ) ( ) ( ) ...
1! 2!
( ) ( )
( ) ( )
! ( 1)!
n n
n n
n n
f x P x R x
f c f c
f c x c x c
f c f c
x c x c
n n
+
+
= +
′ ′′
= + − + − + +
+ − + −
+
trong đó c là một số nằm giữa x và c, định lí được chứng minh.
Người ta thường gọi công thức (4.6.1) là công thức Taylor và biểu diễn một hàm số f(x)
dưới dạng (4.6.1) là khai triển Taylor của hàm số f(x) tại điểm x = c.
4.8.2 Khai triển Maclaurin
Ta thấy khi c = 0, thì (4.6.1) có dạng
23
23
( ) ( 1)
2 1(0) (0) (0) ( )
( ) (0) ...
1! 2! ! ( 1)!
n n
n nf f f f c
f x f x x x x
n n
+
+′ ′′
= + + + + +
+
(4.6.11)
trong đó c nằm giữa x và 0.
Đặt c x= θ , trong đó 0 1θ< < , khi đó (4.6.11) trở thành
( ) ( 1)
2 1(0) (0) (0) ( )
( ) (0) ...
1! 2! ! ( 1)!
n n
n nf f f f x
f x f x x x x
n n
θ+
+′ ′′
= + + + + +
+
(4.6.12)
Công thức (4.6.12) gọi là khai triển Maclaurin của hàm f, trong đó
1
1( )
R ( ) .
( 1)!
n
n
n
f x
x x
n
θ+
+
=
+
Chú ý:
a) Trong định lý Taylor ta thường viết x = c + h, khi đó công thức (4.6.1) có dạng
2
( ) ( 1)
1
( ) ( )
( ) ( ) ...
1! 2!
( ) ( )
.
! ( 1)!
n n
n n
f c f c
f c h f c h h
f c f c h
h h
n n
θ+
+
′ ′′
+ = + + + +
+
+ +
+
(4.6.13)
b) Bây giờ trong công thức trên, nếu thay h bởi dx và nhớ rằng
2 2 ( )
( ) ( ), ( ) ( ),..., ( ) ( )n n n
f c dx df c f c dx d f c f c dx d f c′ ′′= = =
và ( 1) 1 1
( ) ( )n n n
f dx d fξ ξ+ + +
= , ta có thể biểu diễn khai triển trên dưới dạng
2 11 1 1
( ) ( ) ( ) ... ( ) ( )
2! ! ( 1)!
n n
f c df c d f c d f c d f
n n
ξ+
Δ = + + + +
+
(4.6.11)
trong đó . , 0 1c hξ θ θ= + < < .
Ví dụ 1: Trước hết hãy xét f(x) = ex
với x∈
Khi đó ta có:
( ) *
( ) 0 ( ) *
( ) .
(0) 1, ( ) .
k x
k k x
f x e k
f e f x e kθ
θ
= ∀ ∈
= = = ∀ ∈
Theo công thức (4.6.12)
2 1
1 ...
1! 2! ! ( 1)!
n n
x xx x x x
e e
n n
θ
+
= + + + + +
+
với x∈ và 0 1θ< <
hay
2
1 ... ( )
1! 2! !
n
x nx x x
e x
n
ο= + + + + + với x∈ . (4.6.15)
Ví dụ 2: Xét hàm số f(x) = sin x với x∈ .
Ta thấy ( )
( ) sin( )
2
k k
f x x
π
= + với k ∈ , do đó
24
(2 )
(0) 0, (0) sin 0n
f f nπ= = =
(2 1) 1
11
( 1)
(0) sin( ) ( 1) ( 1,2,3,...)
2
| ( )| ( ) .
( 1)! ( 1)!
m m
nn
n
n
f m m
xx
R x f x
n n
π
π
θ
− −
++
+
= − = − =
= ≤
+ +
Vì vậy bằng cách đặt trong công thức (4.6.12) n = 2m, ta được
3 5 2 1
1
2sin ... ( 1)
1! 3! 5! (2 1)!
m
m
m
x x x x
x R
m
−
−
= − + + + − +
−
(4.6.16)
trong đó
2
2 ( )
(2 )!
m
m
x
R x
m
≤
hay
3 2 1
1 2
sin ... ( 1) ( )
3! (2 1)!
n
n nx x
x x x
n
−
−
= − + + − + ο
−
.
Ví dụ 3: Tương tự ta có
2 2
2 1
cos 1 ... ( 1) ( )
2! (2 )!
n
n nx x
x x
n
ο +
= − + + − + . (4.6.17)
trong đó
2 1
2 1
( ) cos
(2 1)!
n
n x
x x
n
θ
+
+
ο =
+
với 0 1θ< < .
Ví dụ 4: Xét khai triển của hàm
1
( )
1
f x
x
=
+
với 1x ≠ −
Ta thấy
( )
1
1 !
( 1)
1 (1 )
n
n
n
n
x x +
⎛ ⎞⎟⎜ = −⎟⎜ ⎟⎟⎜⎝ ⎠+ +
, từ đó ta có
2 ( )
(0) 1, (0) 1, (0) ( 1) 2!,..., (0) ( 1) !n n
f f f f n′ ′′= = − = − = −
Sử dụng công thức (4.6.12) ta được:
21
1 ... ( 1) ( )
1
n n n
x x x x
x
ο= − + − + − +
+
(4.6.18)
trong đó 1 1
1
1
( ) ( 1)
(1 )
n n n
n
x x
xθ
+ +
+
ο = −
+
.
Ví dụ 5: Khai triển f(x) = ln(1+x) với x>−1
Ta nói rằng
1
1
1 !
(0) 0, ( ) , ( ) ( 1)
1 (1 )
n n
n
n
f f x f x
x x
+
+
′= = = −
+ +
Do đó
25
25
2 3
1
ln(1 ) ... ( 1) ( )
2 3
n
n nx x x
x x x
n
−
+ = − + − + − + ο
trong đó 1
1
1 1
( ) ( 1) .
1 (1 )
n n n
n
x x
n xθ
+
+
ο = −
+ +
.
Ví dụ 6: Xét khai triển ( ) (1 ) , , 0f x x α
α α= + ∈ ≠ .
Tương tự như trên, ta có thể chứng minh được rằng
2( 1)
(1 ) 1 ...
2!
( 1)...( 1)
( ).
!
n n
x x x
n
x x
n
α α α
α
α α α
−
+ = + + + +
− − +
+ + ο
(4.6.20)
4.7 Qui tắc L’hospital để khử dạng vô định
Trước hết ta xét giới hạn
( )
lim
( )x c
f x
g x→
trong trường hợp f(x) và g(x) dần tới 0 khi x c→ .
Trường hợp đặc biệt của giới hạn này là:
Nếu như f liên tục tại điểm x0, tức là
0 0
0 0lim( ( ) ( )) lim( ) 0
x x x x
f x f x x x
→ →
− = − =
thì ta gọi giới hạn
0
0
0
( ) ( )
lim
x x
f x f x
x x→
−
−
, nếu tồn tại, là đạo hàm 0( )f x′ . Cho nên ta hy vọng rằng
bằng cách sử dụng những định lý về đạo hàm ta có thể khử được một số dạng vô định của giới
hạn. Các quy tắc sau đây gọi chung là quy tắc L’Hospital.
4.8.1 Dạng vô định
0
0
Hãy xét dạng vô định
( )
lim
( )x c
f x
g x→
, trong đó lim ( ) 0 lim ( )
x c x c
f x g x
→ →
= = ,
với c có thể vô hạn hoặc hữu hạn.
Định lý 4.7.1 Giả sử
i) f và g là các hàm liên tục trên [a,b] và ( , )c a b∈ sao cho f(c) = g(c) = 0
ii) Nếu trong một lân cận nào đó của điểm c (có thể trừ điểm c) tồn tại các đạo hàm ,f g′ ′
với ( ) 0g x′ ≠ .
iii) Ngoài ra tồn tại
( )
lim
( )x c
f x
g x→
′
′
hữu hạn.
Khi đó
( ) ( )
lim lim
( ) ( )x c x c
f x f x
g x g x→ →
′
=
′
. (4.7.1)
Chứng minh: Theo định lí Cauchy
26
( ) ( ) ( ) ( )
( ) ( ) ( ) ( )
f x f x f c f
g x g x g c g
ξ
ξ
′−
= =
′−
trong đó ξ nằm giữa c và x
Khi x c→ thì cξ → , suy ra
( ) ( ) ( )
lim lim lim
( ) ( ) ( )x c c x c
f x f f x
g x g g xξ
ξ
ξ→ → →
′ ′
= =
′ ′
.
Chú ý: Nếu cả hai đạo hàm f ′ và g′ vẫn tiến tới 0 khi x c→ và chúng là các hàm khả vi
trong lân cận của c thì ta có thể áp dụng quy tắc L’Hospital một lần nữa, một cách tổng quát:
( )
( )
( ) ( ) ( )
lim lim ... lim
( ) ( ) ( )
n
nx c x c x c
f x f x f x
g x g x g x→ → →
′′
= = =
′′
.
Ví dụ 1:
0 0
lim lim 2
1ln( 1)
1
x x x x
x x
e e e e
x
x
− −
→ →
− +
= =
+
+
Ví dụ 2:
2
20 0
sin (cos 1) 1
lim lim
arcsin 1 1x x
x x x x
x x x→ →
− − −
= =
− − −
2
20 0
2
2
0 0
(cos 1)
lim 1 lim
1 1
sin 1
lim 1 lim 1.
x x
x x
x
x
x
x x
x
x
→ →
→ →
−
= − =
− −
− −
= − = −
Chú ý: quy tắc được phát biểu trên vẫn đúng cho cả giới hạn phải, giới hạn trái mà việc
chứng minh được làm tương tự.
Ví dụ 3: Tìm giới hạn
2
2
cos
lim
( )
2
x
x
I
x
π π→
=
−
Trước hết ta thấy theo quy tắc L’Hospital
2
sin
lim
2( )
2
x
x
I
x
π π→
−
=
−
.
Mặt khác
2 2
sin sin
lim , lim
2( ) 2( )
2 2
x x
x x
x x
π ππ π− +
→ →
− −
= +∞ = −∞
− −
,
suy ra giới hạn I không tồn tại.
Quy tắc L’Hospital còn đúng cho trường hợp ( ) 0,f x → ( ) 0g x → khi x → +∞ hay
.x → −∞ Ta xét trường hợp khi x → +∞ .
Định lí 4.7.2 Giả sử f và g là những hàm xác định trên (a,+∞ ) sao cho:
i) lim ( ) lim ( ) 0,
x x
f x g x
→+∞ →+∞
= =
ii) f và g khả vi trên (a,+∞ ) và ( ) 0g x′ ≠ khi x đủ lớn,
27
27
iii)
( )
lim
( )x
f x
A
g x→+∞
′
=
′
.
Khi đó
( )
lim
( )x
f x
A
g x→+∞
= . (4.7.2)
Chứng minh: Đặt
1
x
y
= , ta thấy khi x → +∞ thì 0y +
→
Ta có
1
( )
( )
1( ) ( )
f
f x y
g x g
y
= .
Nếu đặt 1 1
1 1
( ) ( ), ( ) ( )f y f g y g
y y
= = thì 1
1
( )( )
.
( ) ( )
f yf x
g x g y
=
Suy ra 1 1
0 0
1 1
( ) ( )( )
lim lim lim
( ) ( ) ( )x y y
f y f yf x
g x g y g y+ +→+∞ → →
′
= = =
′
2
0 0
2
1 1
( )( )
( )
lim lim
1 1 ( )
( )( )
y y
f
y f xy
A
g x
g
y y
+ +
→ →
′ −
′
= = =
′
′ −
,
Do đó
( )
lim
( )x
f x
A
g x→+∞
= , điều phải chứng minh.
4.8.2 Dạng vô dịnh
∞
∞
Định lí 4.7.3 Giả sử
i) Điểm ( , )c a b∈ và trong lân cận nào đó của điểm c (trừ điểm c) các hàm f, g khả vi và
( ) 0g x′ ≠ .
ii) lim| ( )|
x c
f x
→
= +∞ và lim| ( )|
x c
g x
→
= +∞ .
iii)
( )
lim
( )x c
f x
A
g x→
′
=
′
.
Khi đó
( )
lim
( )x c
f x
A
g x→
= . (4.7.3)
Chứng minh:
Do ( , )c a b∈ và
( )
lim
( )x c
f x
A
g x→
′
=
′
nên với 0ε > tuỳ ý cho trước, ta có thể tìm được một số
0δ > sao cho
( )
( )
f t
A A
g t
ε ε
′
− < < +
′
khi t cδ δ− < − < hay khi c t cδ δ− < < +
28
Chọn 0x c δ= + thì
( )
( )
f t
A A
g t
ε ε
′
− < < +
′
khi 0c t x< < . (4.7.4)
Mặt khác, với ( , )x a b∈ sao cho 0c x x< < , khi áp dụng định lí Cauchy ta có
0
0
( ) ( ) ( )
( ) ( ) ( )
f x f x f
g x g x g
ξ
ξ
′−
=
′−
với 0x xξ< < . (4.7.5)
Hơn nữa
0
0
00
( )
1
( ) ( ) ( ) ( )
.
( )( ) ( ) ( )
1
( )
f x
f x f x f x f x
g xg x g x g x
g x
−
−
=
−
−
,
suy ra
0
0
00
( )
1
( ) ( )( ) ( )
.
( )( ) ( ) ( )
1
( )
g x
f x f xf x g x
f xg x g x g x
f x
−
−
=
−
−
. (4.7.6)
Từ (4.7.5) và (4.7.6) ta thấy
0
0
( )
1
( ) ( ) ( )
.
( )( ) ( )
1
( )
g x
f x f g x
f xg x g
f x
ξ
ξ
−
′
=
′
−
. (4.7.7)
Vì 0c x xξ< < < nên
( )
(
f
A A
g
ξ
ε ε
ξ
′
− < < +
′
(4.7.8)
Bây giờ cho x c→ , theo giả thiết lim| ( )| ,
x c
f x
→
= +∞ lim| ( )|
x c
g x
→
= +∞
suy ra
0
0
( )
1
( )
lim 1,
( )
1
( )
x c
g x
g x
f x
f x
→
−
=
−
(4.7.9)
nên ta có thể viết
0
0
( )
1
( )
1 ( ),
( )
1
( )
g x
g x
x
f x
f x
δ
−
= +
−
(4.7.10)
trong đó ( ) 0xδ → khi x c→
Theo hệ thức (4.7.7), ta có
29
29
( )
( )(1 ( )) ( )(1 ( ))
( )
f x
A x A x
g x
ε δ ε δ− + < < + + . (4.7.11)
Do ε nhỏ tuỳ ý, nên
( )
lim
( )x c
f x
A
g x→
= . Định lí được chứng minh.
Ví dụ 4:
2 20 0 0
ln 1 ln
lim lim lim
cot1 cot 1x x x
x x
gxx gx x
+ + +
→ → →
= =
+ +
2
2 20 0 0 0
2
1
1 1 sin
lim . lim lim . lim 0.
11 1
sin
x x x x
xx
xx x
x
+ + + +
→ → → →
= = =
+ +−
Chú ý: Quy tắc L’Hospital có thể dùng để khử các dạng vô định khác 0
;0. ; ;1∞
∞ − ∞ ∞ ∞ .
Ví dụ 5: Tính
0
lim ln
x
I x xα
+
→
= do 0α > .
1
0 0 0
1
ln 1
lim lim lim 0
x x x
x xI x
x x
α
α α
αα+ + +− − −
→ → →
= = = − =
−
do 0α > .
Ví dụ 6: Tính 2 20
1 1
lim( )
sinx
I
x x→
= −
2 2
2 2 2 20 0
sin 2sin cos 2
lim lim
sin 2 sin 2 sin cosx x
x x x x x
x x x x x x x→ →
− −
= =
+
2 20
sin 2 2
lim
2 sin sin 2x
x x
x x x x→
−
=
+
2 20
2
2 2
2 os2 2
lim
2sin 4 sin 2 2 cos2
2sin
lim
sin 2 sin 2 cos2
x
c x
I
x x x x x
x
x x x x x
→
−
=
+ +
= −
+ +
20
2 2
2 1
lim
3sin 2 cos2
1 2
sin sin
x x x x
x
x x
→
= − = −
+ +
.
Ví dụ 7:
ln 0
0 0
) lim lim 1,x x x
x x
a x e e+ +
→ →
= = =
1 ln
1 1
1 1
) lim lim .
x
x x
x x
b x e− −
→ →
=
Ta thấy
1 1
1
ln
lim lim 1
1 1x x
x x
x→ →
= =
−
, nên
1
1
1
lim x
x
x e−
→
= .
30
Cuối cùng ta hãy xét độ tăng của các hàm , vµ logx m
aa x x
Ví dụ 8: Cho a>1 và nếu m là số tuỳ ý, thì
lim
x
mx
a
x→+∞
= +∞ (4.7.12)
Do
2 1
2 1ln ln ln ln
1 ...
1! 2! ! ( 1)!
n x n
x n na a a a a
a x x x x
n n
θ +
+
= + + + + +
+
.
Bây giờ ta hãy chọn số tự nhiên n sao cho n>m. Với x>0, từ (4.7.13) ta thấy
ln
,
!
n
x na
a x
n
> hay
ln
!
x n
n m
m
a a
x
nx
−
> .
Từ đây suy ra lim
x
mx
a
x→+∞
= +∞ .
Ví dụ 9: cho a>1 và m>0,
log
lim 0a
mx
x
x→+∞
= (4.7.14)
Thật vậy, đặt log .log ,m y m
a ax a y x m x= ⇒ = = khi x → +∞ thì y → +∞ . Do đó
log 1
,a
m y
x y
mx a
= theo ví dụ trên
log 1
lim lim 0a
m yx y
x y
mx a→+∞ →+∞
= = , điều phải chứng minh.
Từ các ví dụ trên ta thấy khi x → +∞ , hàm ax
với a>1 tăng nhanh hơn bất cứ hàm luỹ thừa
nào của x. Khi x → +∞ , hàm log , 1a x a > tăng chậm hơn bất kỳ hàm luỹ thừa xm
với số mũ
dương.
4.8 Khảo sát hàm số
4.8.1 Khảo sát đường cong cho dưới dạng phương trình hiện
Xét hàm số ( ), ( , )y f x x a b= ∈ . (4.8.1)
Ở trường phổ thông, để khảo sát sự biến thiên của hàm số ta thường tìm cực đại, cực tiểu
của hàm số theo qui tắc I và quy tắc II và tìm điểm uốn của đồ thị.
Ví dụ 1: Ta hãy xét hàm số 2
( )
1
x
f x
x
=
+
.
Miền xác định của hàm số là ( , )Df = −∞ +∞ .
Ta có
2 2
2 2 2 3
1 2 ( 3)
( ) , ( )
( 1) ( 1)
x x x
f x f x
x x
− −
′ ′′= =
+ +
.
Theo dấu của đạo hàm f’(x) ta thấy rằng hàm số tăng trong khoảng (−1,1), giảm trong
các khoảng ( ,1−∞ ) và (1,+∞ ). Do đó tại điểm –1 hàm số đạt cực tiểu, tại điểm 1 hàm số đạt
cực đại:
31
31
1 1
( 1) , (1)
2 2
f f− = − = .
Đạo hàm cấp hai âm trong khoảng ( , 3−∞ ) và trong khoảng (0, 3 ), dương trong
khoảng (– 3 ,0) và ( 3 ,+∞).
Do đó những điểm (−
3
3,
4
− ); (0,0); ( 3 ,
3
4
) là những điểm uốn của đồ thị. Ngoài ra
ta chú ý rằng
2 2
lim 0 vµ lim 0
1 1x x
x x
x x→+∞ →−∞
= =
+ +
.
Sau đây chúng ta sẽ giới thiệu hai định lí khi f(x) có đạo hàm cấp cao mà phần chứng
minh của nó độc giả có thể đọc trong cuốn [1].
Định lí 4.8.1 Giả sử trong lân cận của điểm x = c hàm f(x) có đạo hàm cấp n và đạo hàm cấp n
tại điểm x = c liên tục. Ngoài ra giả sử
( ) ( )
( ) 0 ví i 1 vµ ( ) 0.k n
f c k n f c= ≤ < ≠ Khi đó
i) Nếu n chẵn, ( )
( ) 0n
f c > , thì hàm f(x) đạt cực tiểu địa phương tại x = c
ii) Nếu n chẵn, ( )
( ) 0n
f c < , thì hàm f(x) đạt cực đại địa phương tại x = c
iii) Nếu n lẻ thì hàm f(x) không đạt cực trị tại x = c.
Định lí 4.8.2 Giả sử trong lân cận điểm x = d hàm f(x) có đạo hàm cấp m và đạo hàm cấp m
tại điểm x = d liên tục. Ngoài ra giả sử ( ) ( )
( ) 0 ví i 1 , nh- ng ( ) 0k m
f d k m f d= ≤ < ≠ . Khi đó
nếu m lẻ thì (d, f(d)) là điểm uốn của đồ thị.
Kết hợp hai định lí trên ta có điều cần nhớ sau đây:
Nếu 0 0( ) 0, ( ) 0f x f x′ ′′= ≠ thì hàm số đạt cực trị tại điểm x = x0 (đạt cực tiểu nếu
0( )f x′′ >0, cực đại nếu 0( )f x′′ <0
Nếu 0( )f x′′ = 0, (3)
0( ) 0f x ≠ thì điểm (x0, f(x0)) là điểm uốn.
Ví dụ 2 Hãy tìm cực trị địa phương và điểm uốn của hàm số
7 6 5 41 1 1 1
( )
7 6 5 4
f x x x x x= + − − .
Ta có
6 5 4 3 3 3 2 3 2
2 2 2
1 2
1 1 1
1 6 3 6 1
( ) ( ) ( ) ( )
( ) ( )( ) ( )( )( ),
f x x x x x x x x x x x x
f x x x x x x x x xα α
′ = + − − = + − − = + −
′′ = + − − = + − −
trong đó 1 2
1 1
1 73 1 73
12 12
( ), ( ).α α= + = −
Do 1 2
5 10 8 7
12 12 12 12
;α α< < − < < − , cho nên 2 11 0 1.α α− < < < <
Ngoài ra (3) 4 3 2
( ) 30 20 12 6 .f x x x x x= + − −
32
Phương trình ( ) 0′ =f x có nghiệm x= 0, –1, 1 và phương trình ( ) 0′′ =f x có nghiệm x = 0,
–1, 1 2,α α .
Ta thấy 3 4
0 0 0 0 0 0( )
( ) ( ) ( ) , ( ) ;f f f f′ ′′= = = <
3
1 0 1 0( )
( ) , ( ) ;f f′′ − = − >
1 0 1 0( ) , ( ) .f f′ ′′= >
Cho nên hàm số đạt cực đại tại x = 0, cực tiểu tại x = 1, điểm uốn tại x = –1, 1 2, .α α
Chú ý đối với hàm số này ta có thể tìm cực đại, cực tiểu theo qui tắc I và xét dấu đạo
hàm cấp hai để tìm điểm uốn.
4.8.2 Đường cong cho dưới dạng tham số
Cho hệ hai phương trình
( )
, ( , )
( )
x x t
t
y y t
α β
=⎧
∈⎨
=⎩
. (4.8.2)
Khi đó với mỗi giá trị ( , )α β∈t hệ phương trình (4.8.2) cho ta một điểm M(x,y) tương
ứng trong mặt phẳng Oxy và khi t biến thiên trong ( , )α β điểm M vạch nên một đường cong
Γ nào đó trong mặt phẳng, vì thế ta gọi hệ phương trình (4.8.2) là hệ phương trình tham số
của đường cong Γ , trong đó t là tham số.
Ví dụ 3: Phương trình tham số của đường thẳng đi qua hai điểm A(a,c) và B(b,d) là
,
x mt a
t
y nt b
= +⎧
∈⎨
= +⎩
(4.8.3)
trong đó m = b − a, n = d − c.
Ví dụ 4: Phương trình tham số của ellip
2 2
2 2
1
x y
a b
+ = là
0 2
cos
ví i [ , ]
sin
x a t
t
y b t
π
=⎧
∈⎨
=⎩
. (4.8.4)
Để khảo sát đường cong cho dưới dạng tham số ta cần thực hiện các bước sau đây:
a) Tìm tập xác định của các hàm số x = x(t), y= y(t)
b) Xét chiều biến thiên của x, y theo t
c) Tìm các đường tiệm cận:
i ) Nếu
0
( )
lim
t t
t
y
→
→±∞
= ±∞ và
0
( )
lim
t t
t
x a
→
→±∞
= (hữu hạn)
thì đường cong có tiệm cận đứng là x = a.
ii) Nếu
0
( )
lim
t t
t
x
→
→±∞
= ±∞ và
0
( )
lim
t t
t
y b
→
→±∞
= (hữu hạn)
thì đường cong đó có tiệm cận ngang là y = b.
33
33
iii) Nếu
0
( )
lim
t t
t
x
→
→±∞
= ∞ ,
0
( )
lim
t t
t
y
→
→±∞
= ∞ , và nếu
0 0
( ) ( )
lim , lim ( )
t t t t
t t
y
a b y ax
x→ →
→±∞ →±∞
= = −
thì đường cong có tiệm xiên là y = ax+b.
Ví dụ 5: Khảo sát và vẽ đường cong axtrôit
2 2 2
3 3 3
0,x y a a+ = > . Dễ thấy phương trình tham số của đường cong nói trên là
3
3
0
cos
, ( , ),
sin
x a t
t a
y a t
⎧ =⎪
∈ −∞ +∞ >⎨
=⎪⎩
. (4.8.5)
Trước hết, ta thấy đường cong y không có tiệm cận. Hơn nữa x,y là các hàm tuần hoàn
với chu kỳ 2π nên ta chỉ cần khảo sát đường cong đã cho trong đoạn [0,2 ]π .
2
2
3
3 0 0 2
2 2
3
3 0 0 2
2 2
( ) cos .sin khi ; ; ; ;
( ) sin .cos khi ; ; ; ; .
x t a t t t
y t a t t t
π π
π π
π π
π π
′ = − = =
′ = = =
Cuối cùng ta nhận xét rằng đối với đường cong axtrôit ta có
2
2
3
tg
3
sin cos
cos sin
t
t
ydy a t t
t
dx x a t t
′
= = = −
′ −
.
Do đó 0
dy
dx
= tại 0 2; ;t π π= và tại các điểm này tiếp tuyến thẳng đứng (xem hình 4.8.1).
34
Hình 4.8.1
Ví dụ 6: Khảo sát và vẽ đường cong cho bởi phương trình
3 3
3 0 0,x y axy a+ − = > . (4.8.6)
Ta thấy rằng khi thay x bởi y và y bởi x thì phương trình (4.8.6) không thay đổi. Do đó,
đồ thị của nó đối xứng nhau qua đường phân giác thứ nhất.
Ta đặt y = tx và thay vào phương trình trên ta được 3 3 3 2
3 0x x t ax t+ − = , từ đây suy ra
2
3 3
3 3
1 1
,
at at
x y
t t
= =
+ +
với 1t ≠ − . (4.8.7)
Lấy vi phân ta thu được
3
3 2 3
1 2 1
( ) 3 ; ( ) 0 khi vµ
(1 ) 2
t
x t a x t t
t
−
′ ′= = =
+
35
35
3
3
3 2
2
( ) 3 ; ( ) 0 khi 2
(1 )
t
y t at y t t
t
−
′ ′= = =
+
.
Khi 1t → − thì x, y đều dẫn tới ∞ và
1 1
lim lim 1
t t
y
a t
x→− →−
= = = −
3
3 31 1
3 3
lim( ) lim[ ]
1 1t t
at at
b y ax
t t→− →−
= − = + =
+ +
2
3 21 1
1 ( 1)
3 lim 3 lim
1 ( 1)( 1)t t
t t t
a a a
t t t t→− →−
+ +
= = = −
+ + − +
.
Vậy tiệm cận xiên của đường cong là y = − x − a. Từ các kết quả trên ta có bảng biến
thiên.
Cuối cùng ta hãy chú ý thêm rằng
3
3
(2 )
1 2
dy t t
dx t
−
=
−
Do đó 3
3
1
0 khi 0, 2, khi ,
2
dy dy
t t t t
dx dx
= = = = ∞ = ∞ = .
Các tiếp tuyến của đường cong ứng với hai giá trị = 0,t = 3
2t song song với trục Ox.
Các tiếp tuyến ứng với hai giá trị = ∞ = 3
1
vµ
2
t t song song với trục Oy (xem hình 4.8.2)
36
3 3
3 0x y axy+ − =
y x a= − x
Hình 4.8.2 Hình 4.8.3
4.8.3 Khảo sát đường cong trong tọa độ cực
a) Hệ tọa độ cực
Trong mặt phẳng chọn một điểm O cố định và tia Ox đi qua điểm O. Ta gọi điểm O là
cực, tia Ox gọi là trục cực. Hệ tọa độ xác định bởi cực và trục cực gọi là hệ tọa độ cực. Gọi
OP là véc tơ đơn vị nằm trên tia Ox. Vị trí của điểm M trong mặt phẳng được xác định bởi
véc tơ OM , nghĩa là xác định bởi góc ϕ = ( , )OP OM và =| |r OM , ϕ được gọi là góc cực, r
được gọi là bán kính cực. Góc gọi là góc định hướng, lấy giá trị dương nếu chiều quay OP
đến trùng với OM ngược chiều kim đồng hồ và lấy giá trị âm nếu ngược lại.
Cặp số có thứ tự ϕ( , )r gọi là tọa độ của điểm M trong mặt phẳng.
b) Mối liên hệ giữa tọa độ Descartes vuông góc và tọa độ cực
Bây giờ ta lấy trục hoành trùng với trục cực và trục tung ứng với tia
π
ϕ =
2
ta được hệ tọa
độ Descartes vuông góc.
Gọi (x,y) và ϕ( , )r lần lượt là tọa độ của điểm M nói trên trong hệ tọa độ Descartes vuông
góc và hệ tọa độ cực. Khi đó, ta có
cos
ví i 0 2 , 0
sin
x r
r
y r
ϕ
ϕ π
ϕ
=⎧
≤ < ≥⎨
=⎩
Cho hàm số ( )r f ϕ= . Trước khi khảo sát hàm số ta có nhận xét sau. Giả sử cho điểm
M(x,y) nằm trên đồ thị. Gọi β là góc dương giữa véctơ OM và véctơ chỉ phương của tiếp
tuyến với đồ thị tại điểm M. Gọi là góc dương giữa trục cực và tiếp tuyến, ta có.
α ϕ
β α ϕ β
α ϕ
−
= − =
+
tg tg
, tg
1 tg tg
. (4.8.8)
Mặt khác theo ý nghĩa hình học của đạo hàm
37
37
'sin cos
tg
'cos sin
dy r r
dx r r
ϕ ϕ
α
ϕ ϕ
+
= =
−
,
trong đó
'
dr
r
dϕ
= (4.8.9)
Thay vào (4.8.9) vào (4.8.8) ta được
Hình 4.8.4
'sin cos sin
'cos sin cos
tg
'sin cos sin
1
'cos sin cos
r r
r r
r r
r r
ϕ ϕ ϕ
ϕ ϕ ϕ
β
ϕ ϕ ϕ
ϕ ϕ ϕ
+
−
−
=
+
+
−
Sau các phép biến đổi đơn giản tg
'
r
r
β = . (4.8.10)
Ví dụ 7: Hãy vẽ đường xoắn ốc lôgarit có phương trình:
, 0, 0b
r ae a bϕ
= > > . (4.8.11)
Ta thấy hàm số r xác định với mọi ϕ . Khi ϕ tăng r cũng tăng, khi 0ϕ = thì r = a, khi
ϕ → +∞ thì r → +∞ , khi ϕ → −∞ thì 0r → và khi đó đường cong quấn vô hạn quanh cực O;
O được gọi là điểm tiệm cận của đường cong. Theo công thức (4.8.9) ta có
1
tg
'
r
r a
β = = ,
Do đó véc tơ chỉ phương của tiếp tuyến với đường cong luôn luôn tạo với OM một góc
không đổi (xem hình 4.8.5).
0
β
ϕ
α
H×nh 4.8.4
38
β
Hình 4.8.5
Ví dụ 8: Hãy vẽ đường hoa hồng ba cánh có phương trình sin 3 , 0r a aϕ= > . Ở đây r là một
hàm tuần hoàn với chu kì
2
3
π
vì thế chỉ cần khảo sát hàm số trong một đoạn có độ dài bằng
chu kì, chẳng hạn đoạn [0,
2
3
π
].
Ta có ' 3 cos3 ; ' 0 khi , ,
6 2
r a r
π π
ϕ ϕ ϕ= = = =
1
tg tg3
' 3
r
r
β ϕ= = .
Đồ thị ứng với khoảng [0,
2
3
π
] gồm hai cánh, sau đo cho đồ thị quay các góc quanh cực ta sẽ
có toàn bộ đồ thị (xem hình 4.8.6)
Hình 4.8.6
39
39
4.9 Bài tập chương 4
4.1 Cho hàm 1
( )
1 x
x
f x
e
=
+
với 0x ≠ và f(0)=0. Chứng minh rằng hàm f(x) liên tục tại
mọi điểm, nhưng (0) 0,f+
′ = (0) 1f−
′ = .
4.2 Chứng minh rằng hàm
1
( ) sinf x x
x
= với 0x ≠ , và f(0)=0 liên tục tại x = 0, nhưng
không có đạo hàm bên trái và bên phải tại x = 0.
4.3 Chứng minh rằng hàm
2
nÕu lµ h÷u tØ
( )
0 nÕu v« tØ
x x
f x
x
⎧⎪
= ⎨
⎪⎩
chỉ có đạo hàm tại x=0.
4.4 Dựa vào định nghĩa hãy tính f’(a) nếu
( ) ( ) ( ),f x x a xϕ= −
trong đó ( )xϕ liên tục tại x=a.
4.5 Cho hàm y=sgnx được định nghĩa sau
1 nÕu 0,
sgn 0 nÕu 0,
1 nÕu 0.
x
x x
x
− <⎧
⎪
= =⎨
⎪ >⎩
Chứng minh rằng |x|=xsgnx
4.6 Tính đạo hàm của các hàm số sau
1) y=|x|
2) y=x|x|
3) y=ln|x| với 0x ≠ .
4.7 Tính đạo hàm các hàm sau
1) y x x x= + +
2) 2 3
| ( 1) ( 1) |y x x= − + .
4.8 Tính y’ nếu
1) y=f(x2
)
2) y=f(sin2
x)+ f(cos2
x)
3) y=f(f(f(x))), trong đó f(x) là hàm khả vi.
40
4.9 Chứng minh rằng hàm
2 1
sin nÕu 0
( )
0 nÕu 0
x x
f x x
x
⎧
≠⎪
= ⎨
⎪ =⎩
có đạo hàm gián đoạn.
4.10 Tính đạo hàm các hàm sau
1) y=xx
2) y=xslnx
, trong đó s là hằng số.
4.11 Tính đạo hàm y’x nếu hàm số y được cho dưới dạng
2 2 2 2
cos , sint t
x e t y e t= = .
4.12 Xác định miền tồn tại hàm ngược x = x(y) và tìm đạo hàm của nó nếu
1) y=x+lnx (x>0) 2) y=x+ex
4.13 Đưa về dạng F(x,y) = 0 (hay y = f(x)) phương trình các đường cong cho dưới dạng
tham số.
1) x = acost, y = bsint 2) x = acos3
t, y = asin3
t
3) ,
2 2
t t t t
e e e e
x y
− −
+ −
= = 4) x = tgt, y = cos2
t.
4.14 Cho hàm f(x)=x3
−2x+1. Hãy xác định (1)fΔ và (1)df nếu 0,1xΔ = .
4.15 Tìm vi phân của hàm
1) ( arctg )
a x
d
x a
+ , a là hằng số
2) (1 cos )d u−
3) ( )bt
d bt e−
− , b là hằng số.
4.16 Hãy tính
1) 3 6 9
2
( 2 )
( )
d
x x x
d x
− −
2) 2
sin
( )
( )
d x
xd x
3)
(tg )
(cot g )
d x
d x
4.17 Hãy tìm hàm ( , )x xθ θ= Δ sao cho ( ) ( )f x x f x+ Δ − = = '( ),xf x xθΔ + Δ (0 1)θ< < nếu
1) 2
( ) ( 0)f x ax bx c a= + + ≠
2) ( ) x
f x e= .
41
41
4.18 Cho hàm (1)
( , )f C∈ −∞ +∞ và hệ thức ( ) ( ) . '( )f x h f x h f x+ − = được nghiệm đúng
,x h R∀ ∈ .
Chứng minh rằng
( )f x ax b= + trong đó a,b là hằng số.
4.19 Cho hàm số 3
( ) , [ 1,1]f x x x= ∈ − . Hỏi có thể tìm được hai số 1 2, ( 1,1)x x ∈ − sao cho
2 1
2 1
( ) ( )
'(0)
f x f x
f
x x
−
=
−
hay không?
4.20 Cho hàm f(x) khả vi trên đoạn 1 2[ , ]x x , trong đó 1 2. 0x x > . Chứng minh rằng
1 2
1 21 2
1
( ) ( )
( ) ( )
x x
f f
f x f xx x
ξ ξ ξ′= −
−
, trong đó 1 2x xξ< < .
4.21 Cho hàm f(x) liên tục trên [a,b] và có đạo hàm tại mọi điểm ( , )x a b∈ . Chứng minh
rằng bằng cách áp dụng định lí Rolle đối với hàm
( ) 1
( ) ( ) 1
( ) 1
x f x
x b f b
a f a
φ = ta sẽ thu được định lí Lagrange
4.22 Cho :f → chứng minh rằng nếu
( 1)
(0) '(0) (0) ... (0) 0n
f f f f −
′′= = = = =
thì
( )
( ) ( )
!
n
n
f x f x
nx
θ
= trong đó 0 1θ< < .
4.23 Tìm đạo hàm cấp 2 của các hàm sau
1) cosx
y e x=
2) 3x
y a x=
3) 2
siny x x= .
4.24 Tìm đạo hàm cấp 3 của các hàm sau
1) sinx
y e x−
=
2) 2
lny x x=
3) cosy x x= .
4.25 Cho ( )
x
af x xe= . Tìm (4) ( ) ( )
( ), ( ), (0)n n
f x f x f
4.26 Cho ( )
1
x
f x
x
=
+
. Chứng minh rằng với 2n ≥
( ) 1
1
1.3.5...(2 3)
(0) ( 1)
2
n n
n
n
f n−
−
−
= − .
42
4.27 Cho 2
1
( )
1
f x
x
=
−
. Chứng minh rằng
( ) ! khi 2
(0)
0 khi 2 1.
n n n m
f
n m
=⎧
= ⎨
= −⎩
4.28 Cho hàm số 2
( ) , 0
x
af x x e a
−
= ≠ . Chứng minh rằng
( )
2
( 1)( 1)
(0)
n
n
n
n n
f
a −
− −
= .
4.29 1) Cho *
( ) ,n
f x x n N= ∈ . Chứng minh rằng
( )
(1) (1) (1)
(1) ... 2
1! 2! !
n
nf f f
f
n
′ ′′
+ + + + =
2) Cho ( sin ), (1 cos )x a t t y a t= − = − . Tính
2
2
d y
dx
3) Cho = =2 3
, 3t t
x e y . Tính
2
2
d y
dx
.
4.30 Cho đa thức Legendre:
2 ( )1
( ) [( 1) ] ( 0,1,2...)
2 . !
m m
m m
P x x m
m
= − =
Chứng minh rằng ( )mP x thỏa mãn phương trình
2
(1 ) ( ) 2 ( ) ( 1) ( ) 0.m m mx P x xP x m m P x′′ ′− − + + =
4.31 Cho đa thức Lague:
( )
( ) ( )x m x m
mL x e x e−
=
thỏa mãn phương trình
( ) (1 ) ( ) ( ) 0m m mxL x x L x mL x′′ ′+ − + =
4.32 Chứng minh công thức:
1
1
( ln ) !(ln )
n n
n
n
k
d
x x n x
kdx =
= + ∑ với x>0.
4.33 Khai triển hàm
2
2
( ) x x
f x e −
= đến số hạng chứa x5
.
4.34 Khai triển hàm ( )
1x
x
f x
e
=
−
đến số hạng chứa x4
.
4.35 Tìm 3 số hạng đầu tiên của khai triển Taylor trong lân cận của điểm x=0.
1) (sinx)2
2) cos x .
43
43
4.36 Cho n số 1 2, ,..., na a a . Xác định x sao cho hàm số.
2
1
( ) ( )
n
k
k
x a xϕ
=
= −∑ có giá trị bé nhất.
4.37 Tìm 3 số hạng của khai triển hàm ( )f x x= theo các lũy thừa nguyên dương của
hiệu x−1.
4.38 Khai triển hàm f(x)=xx
−1 theo các lũy thừa nguyên dương của nhị thức (x−1) đến số
hạng chứa (x−1)3
.
4.39 Áp dụng qui tắc L’hospital để tìm các giới hạn sau:
1)
1
1 1
lim( )
ln 1x x x→
−
−
2)
2
1
0
sin
lim( )x
x
x
x→
3) 20
2 ln(1 2 )
lim
2x
x x
x→
− +
4)
2
20
1 2
lim
2
x
x
e x
x→
− −
.
4.40 Hãy tìm giới hạn sau, xét xem có thể áp dụng qui tắc L’hospital hay không
1)
2
0
1
sin
lim
sinx
x
x
x→
2)
sin
lim
sinx
x x
x x→∞
−
+
3) sin
1 sin cos
lim
( sin cos ) xx
x x x
x x x e→∞
+ +
+
.
4.41 Tìm các giới hạn sau:
1)
0
1 1
lim( )
1xx x e→
−
−
2)
0
ln(sin )
lim
ln(sin )x
ax
bx→
3)
2
2
40
lim
sin
x
x
e x
x→
−
4)
0
lim(sin )tgx
x
x
→
5)
1
2 1
0
lim(1 )
x
e x
x
x − −
→
+ 6) 2cos
2
lim( ) x
x
tgx
π
→
.
4.42 Chứng minh rằng khi 0x → ta có
1)
3
arctg
3
x
x x− ∼ 2) ~ lnx x a
a b x
b
−
3) 2 2
1 2 ~ 2x
e x x− − 4) 2
2 ln(1 2 ) ~ 2x x x− + .
4.43 Cho hàm 6 1
( ) sing x x
x
= với 0; (0) 0x g≠ = và 6
( ) 2 ( ).f x x g x= +
Xét cực trị địa phương của các hàm g, f tại điểm x=0.
4.44 Khảo sát và vẽ đồ thị hàm số
2
2
( 1)
1
x
y
x
−
=
+
.
44
4.45 Khảo sát và vẽ đồ thị hàm số
ln
.
x
y
x
=
4.46 Khảo sát và vẽ đồ thị hàm số
2 x
y x e−
= .
4.47 Khảo sát và vẽ đồ thị hàm số
2 2
1 1y x x= + + − .
4.48 Khảo sát và vẽ đồ thị hàm số
1 2cosr ϕ= + .

More Related Content

What's hot

11 phuong phap giai pth
11 phuong phap giai pth11 phuong phap giai pth
11 phuong phap giai pthPhuc Nguyen
 
Hướng dẫn giải bài tập chuỗi - Toán cao cấp
Hướng dẫn giải bài tập chuỗi - Toán cao cấpHướng dẫn giải bài tập chuỗi - Toán cao cấp
Hướng dẫn giải bài tập chuỗi - Toán cao cấpVan-Duyet Le
 
[Vnmath.com] 13-ki-thuat-giai-phuong-trinh-ham
[Vnmath.com] 13-ki-thuat-giai-phuong-trinh-ham[Vnmath.com] 13-ki-thuat-giai-phuong-trinh-ham
[Vnmath.com] 13-ki-thuat-giai-phuong-trinh-hamDuy Duy
 
Thamkhao.vn 4607 phuong-phap-giai-he-phuong-trinh-trong-ky-thi-tuyen-sinh-dh
Thamkhao.vn 4607 phuong-phap-giai-he-phuong-trinh-trong-ky-thi-tuyen-sinh-dhThamkhao.vn 4607 phuong-phap-giai-he-phuong-trinh-trong-ky-thi-tuyen-sinh-dh
Thamkhao.vn 4607 phuong-phap-giai-he-phuong-trinh-trong-ky-thi-tuyen-sinh-dhHuynh ICT
 
Cđ van dung bdt giai pt hpt
Cđ van dung bdt giai pt hptCđ van dung bdt giai pt hpt
Cđ van dung bdt giai pt hptCảnh
 
Hệ Hoán Vị Vòng Quanh
Hệ Hoán Vị Vòng QuanhHệ Hoán Vị Vòng Quanh
Hệ Hoán Vị Vòng QuanhNhập Vân Long
 
Ky thuat giai he phuong trinh
Ky thuat giai he phuong trinhKy thuat giai he phuong trinh
Ky thuat giai he phuong trinhHuynh ICT
 
Toan a1 -_bai_giang
Toan a1 -_bai_giangToan a1 -_bai_giang
Toan a1 -_bai_giangxuanhoa88
 
đại số lớp 11
đại số lớp 11đại số lớp 11
đại số lớp 11Luna Trần
 
100 bt về phương trình và hệ pt
100 bt về phương trình và hệ pt100 bt về phương trình và hệ pt
100 bt về phương trình và hệ ptDũng Bùi
 
Chuyên đề hệ phương trình bằng phương pháp hàm số
Chuyên đề hệ phương trình bằng phương pháp hàm sốChuyên đề hệ phương trình bằng phương pháp hàm số
Chuyên đề hệ phương trình bằng phương pháp hàm sốVui Lên Bạn Nhé
 
11 mat102-bai 8-v1.0
11 mat102-bai 8-v1.011 mat102-bai 8-v1.0
11 mat102-bai 8-v1.0Yen Dang
 
Phuong phap tich phan
Phuong phap tich phanPhuong phap tich phan
Phuong phap tich phanphongmathbmt
 
Pp giai pt va hpt khong mau muc
Pp giai pt va hpt khong mau mucPp giai pt va hpt khong mau muc
Pp giai pt va hpt khong mau muckeolac410
 
[Vnmath.com] phuong-trinh-bpt-trong-de-thi-thu-2014
[Vnmath.com] phuong-trinh-bpt-trong-de-thi-thu-2014[Vnmath.com] phuong-trinh-bpt-trong-de-thi-thu-2014
[Vnmath.com] phuong-trinh-bpt-trong-de-thi-thu-2014Antonio Krista
 

What's hot (18)

11 phuong phap giai pth
11 phuong phap giai pth11 phuong phap giai pth
11 phuong phap giai pth
 
Hướng dẫn giải bài tập chuỗi - Toán cao cấp
Hướng dẫn giải bài tập chuỗi - Toán cao cấpHướng dẫn giải bài tập chuỗi - Toán cao cấp
Hướng dẫn giải bài tập chuỗi - Toán cao cấp
 
[Vnmath.com] 13-ki-thuat-giai-phuong-trinh-ham
[Vnmath.com] 13-ki-thuat-giai-phuong-trinh-ham[Vnmath.com] 13-ki-thuat-giai-phuong-trinh-ham
[Vnmath.com] 13-ki-thuat-giai-phuong-trinh-ham
 
Luận văn: Phép biến đổi phân tuyến tính, HAY, 9đ
Luận văn: Phép biến đổi phân tuyến tính, HAY, 9đLuận văn: Phép biến đổi phân tuyến tính, HAY, 9đ
Luận văn: Phép biến đổi phân tuyến tính, HAY, 9đ
 
Thamkhao.vn 4607 phuong-phap-giai-he-phuong-trinh-trong-ky-thi-tuyen-sinh-dh
Thamkhao.vn 4607 phuong-phap-giai-he-phuong-trinh-trong-ky-thi-tuyen-sinh-dhThamkhao.vn 4607 phuong-phap-giai-he-phuong-trinh-trong-ky-thi-tuyen-sinh-dh
Thamkhao.vn 4607 phuong-phap-giai-he-phuong-trinh-trong-ky-thi-tuyen-sinh-dh
 
Cđ van dung bdt giai pt hpt
Cđ van dung bdt giai pt hptCđ van dung bdt giai pt hpt
Cđ van dung bdt giai pt hpt
 
Hệ Hoán Vị Vòng Quanh
Hệ Hoán Vị Vòng QuanhHệ Hoán Vị Vòng Quanh
Hệ Hoán Vị Vòng Quanh
 
Ky thuat giai he phuong trinh
Ky thuat giai he phuong trinhKy thuat giai he phuong trinh
Ky thuat giai he phuong trinh
 
Toan a1 -_bai_giang
Toan a1 -_bai_giangToan a1 -_bai_giang
Toan a1 -_bai_giang
 
đại số lớp 11
đại số lớp 11đại số lớp 11
đại số lớp 11
 
100 bt về phương trình và hệ pt
100 bt về phương trình và hệ pt100 bt về phương trình và hệ pt
100 bt về phương trình và hệ pt
 
Tổng hợp hệ pt
Tổng hợp hệ ptTổng hợp hệ pt
Tổng hợp hệ pt
 
Chuyên đề hệ phương trình bằng phương pháp hàm số
Chuyên đề hệ phương trình bằng phương pháp hàm sốChuyên đề hệ phương trình bằng phương pháp hàm số
Chuyên đề hệ phương trình bằng phương pháp hàm số
 
Chuyen de dao ham
Chuyen de dao ham Chuyen de dao ham
Chuyen de dao ham
 
11 mat102-bai 8-v1.0
11 mat102-bai 8-v1.011 mat102-bai 8-v1.0
11 mat102-bai 8-v1.0
 
Phuong phap tich phan
Phuong phap tich phanPhuong phap tich phan
Phuong phap tich phan
 
Pp giai pt va hpt khong mau muc
Pp giai pt va hpt khong mau mucPp giai pt va hpt khong mau muc
Pp giai pt va hpt khong mau muc
 
[Vnmath.com] phuong-trinh-bpt-trong-de-thi-thu-2014
[Vnmath.com] phuong-trinh-bpt-trong-de-thi-thu-2014[Vnmath.com] phuong-trinh-bpt-trong-de-thi-thu-2014
[Vnmath.com] phuong-trinh-bpt-trong-de-thi-thu-2014
 

Viewers also liked

Giai bai-toan-lien-quan-kshs
Giai bai-toan-lien-quan-kshsGiai bai-toan-lien-quan-kshs
Giai bai-toan-lien-quan-kshsHuynh ICT
 
Khao sat ham so
Khao sat ham soKhao sat ham so
Khao sat ham soHuynh ICT
 
Phuong trinh chua can
Phuong trinh chua canPhuong trinh chua can
Phuong trinh chua canHuynh ICT
 
Phuong trinh luong giac nang cao le van doan ltdh
Phuong trinh luong giac nang cao  le van doan ltdhPhuong trinh luong giac nang cao  le van doan ltdh
Phuong trinh luong giac nang cao le van doan ltdhHuynh ICT
 
Chuyen de-so-phuc-ltdh-nt long- www.mathvn.com
Chuyen de-so-phuc-ltdh-nt long- www.mathvn.comChuyen de-so-phuc-ltdh-nt long- www.mathvn.com
Chuyen de-so-phuc-ltdh-nt long- www.mathvn.comHuynh ICT
 
De bai-toan-lien-quan-khao-sat-ham-so
De bai-toan-lien-quan-khao-sat-ham-soDe bai-toan-lien-quan-khao-sat-ham-so
De bai-toan-lien-quan-khao-sat-ham-soHuynh ICT
 
8.2 pt mat_phang_trong_kg
8.2 pt mat_phang_trong_kg8.2 pt mat_phang_trong_kg
8.2 pt mat_phang_trong_kgHuynh ICT
 
Www.mathvn.com 33 dang toan khao sat ham so ltdh
Www.mathvn.com   33 dang toan khao sat ham so ltdhWww.mathvn.com   33 dang toan khao sat ham so ltdh
Www.mathvn.com 33 dang toan khao sat ham so ltdhHuynh ICT
 
Chuyen de-tich-phan1
Chuyen de-tich-phan1Chuyen de-tich-phan1
Chuyen de-tich-phan1Huynh ICT
 
Chu de cuc tri ham so
Chu de cuc tri ham soChu de cuc tri ham so
Chu de cuc tri ham soHuynh ICT
 
2thi thu dh khoi a vinh phuc lan 1 www.mathvn.com
2thi thu dh khoi a vinh phuc lan 1 www.mathvn.com2thi thu dh khoi a vinh phuc lan 1 www.mathvn.com
2thi thu dh khoi a vinh phuc lan 1 www.mathvn.comHuynh ICT
 
Luonggiac chuong1
Luonggiac chuong1Luonggiac chuong1
Luonggiac chuong1Huynh ICT
 
Chuyen de phuong trinh bac hai doi voi sin va cos
Chuyen de phuong trinh bac hai doi voi sin va cosChuyen de phuong trinh bac hai doi voi sin va cos
Chuyen de phuong trinh bac hai doi voi sin va cosHuynh ICT
 
Cac bai toan lien quan den khao sat hs
Cac bai toan lien quan den khao sat hsCac bai toan lien quan den khao sat hs
Cac bai toan lien quan den khao sat hsHuynh ICT
 
Luonggiac chuong4
Luonggiac chuong4Luonggiac chuong4
Luonggiac chuong4Huynh ICT
 
Cac bai toan co ban lien quan den kshs
Cac bai toan co ban lien quan den kshsCac bai toan co ban lien quan den kshs
Cac bai toan co ban lien quan den kshsHuynh ICT
 

Viewers also liked (18)

Giai bai-toan-lien-quan-kshs
Giai bai-toan-lien-quan-kshsGiai bai-toan-lien-quan-kshs
Giai bai-toan-lien-quan-kshs
 
Khao sat ham so
Khao sat ham soKhao sat ham so
Khao sat ham so
 
Phuong trinh chua can
Phuong trinh chua canPhuong trinh chua can
Phuong trinh chua can
 
Phuong trinh luong giac nang cao le van doan ltdh
Phuong trinh luong giac nang cao  le van doan ltdhPhuong trinh luong giac nang cao  le van doan ltdh
Phuong trinh luong giac nang cao le van doan ltdh
 
Chuyen de-so-phuc-ltdh-nt long- www.mathvn.com
Chuyen de-so-phuc-ltdh-nt long- www.mathvn.comChuyen de-so-phuc-ltdh-nt long- www.mathvn.com
Chuyen de-so-phuc-ltdh-nt long- www.mathvn.com
 
De bai-toan-lien-quan-khao-sat-ham-so
De bai-toan-lien-quan-khao-sat-ham-soDe bai-toan-lien-quan-khao-sat-ham-so
De bai-toan-lien-quan-khao-sat-ham-so
 
8.2 pt mat_phang_trong_kg
8.2 pt mat_phang_trong_kg8.2 pt mat_phang_trong_kg
8.2 pt mat_phang_trong_kg
 
Www.mathvn.com 33 dang toan khao sat ham so ltdh
Www.mathvn.com   33 dang toan khao sat ham so ltdhWww.mathvn.com   33 dang toan khao sat ham so ltdh
Www.mathvn.com 33 dang toan khao sat ham so ltdh
 
Số phức
Số phứcSố phức
Số phức
 
Chuyen de-tich-phan1
Chuyen de-tich-phan1Chuyen de-tich-phan1
Chuyen de-tich-phan1
 
Khao sat hs
Khao sat hsKhao sat hs
Khao sat hs
 
Chu de cuc tri ham so
Chu de cuc tri ham soChu de cuc tri ham so
Chu de cuc tri ham so
 
2thi thu dh khoi a vinh phuc lan 1 www.mathvn.com
2thi thu dh khoi a vinh phuc lan 1 www.mathvn.com2thi thu dh khoi a vinh phuc lan 1 www.mathvn.com
2thi thu dh khoi a vinh phuc lan 1 www.mathvn.com
 
Luonggiac chuong1
Luonggiac chuong1Luonggiac chuong1
Luonggiac chuong1
 
Chuyen de phuong trinh bac hai doi voi sin va cos
Chuyen de phuong trinh bac hai doi voi sin va cosChuyen de phuong trinh bac hai doi voi sin va cos
Chuyen de phuong trinh bac hai doi voi sin va cos
 
Cac bai toan lien quan den khao sat hs
Cac bai toan lien quan den khao sat hsCac bai toan lien quan den khao sat hs
Cac bai toan lien quan den khao sat hs
 
Luonggiac chuong4
Luonggiac chuong4Luonggiac chuong4
Luonggiac chuong4
 
Cac bai toan co ban lien quan den kshs
Cac bai toan co ban lien quan den kshsCac bai toan co ban lien quan den kshs
Cac bai toan co ban lien quan den kshs
 

Similar to Chuong 4 x

Tổng hợp bồi dưỡng học sinh giỏi
Tổng hợp bồi dưỡng học sinh giỏiTổng hợp bồi dưỡng học sinh giỏi
Tổng hợp bồi dưỡng học sinh giỏiNhập Vân Long
 
Cac phuong phap giai pt ham thuong dung
Cac phuong phap giai pt ham thuong dungCac phuong phap giai pt ham thuong dung
Cac phuong phap giai pt ham thuong dungljmonking
 
06 mat101 bai2_v2.3013101225
06 mat101 bai2_v2.301310122506 mat101 bai2_v2.3013101225
06 mat101 bai2_v2.3013101225Yen Dang
 
Luận văn: Vài vấn đề cơ bản của hàm nhiều biến phức, HAY
Luận văn: Vài vấn đề cơ bản của hàm nhiều biến phức, HAYLuận văn: Vài vấn đề cơ bản của hàm nhiều biến phức, HAY
Luận văn: Vài vấn đề cơ bản của hàm nhiều biến phức, HAYViết thuê trọn gói ZALO 0934573149
 
kỹ thuật giải phương trình hàm
kỹ thuật giải phương trình hàmkỹ thuật giải phương trình hàm
kỹ thuật giải phương trình hàmljmonking
 
13 ki-thuat-giai-phuong-trinh-ham (1)
13 ki-thuat-giai-phuong-trinh-ham (1)13 ki-thuat-giai-phuong-trinh-ham (1)
13 ki-thuat-giai-phuong-trinh-ham (1)ljmonking
 
Chứng minh bổ đề lagrange.doc
Chứng minh bổ đề lagrange.docChứng minh bổ đề lagrange.doc
Chứng minh bổ đề lagrange.docHoang Mai
 
[Math educare] giao trinh toan cao cap a1-giai tich ham mot bien_chuoi so
[Math educare] giao trinh toan cao cap a1-giai tich ham mot bien_chuoi so[Math educare] giao trinh toan cao cap a1-giai tich ham mot bien_chuoi so
[Math educare] giao trinh toan cao cap a1-giai tich ham mot bien_chuoi soNguyen Vietnam
 
Tom tat-li-thuyet-va-cong-thuc-giai-nhanh-toan-12-tran-quoc-nghia
Tom tat-li-thuyet-va-cong-thuc-giai-nhanh-toan-12-tran-quoc-nghiaTom tat-li-thuyet-va-cong-thuc-giai-nhanh-toan-12-tran-quoc-nghia
Tom tat-li-thuyet-va-cong-thuc-giai-nhanh-toan-12-tran-quoc-nghiaBlogTi
 
giai-tich-2__kha-vi-va-vi-phan - [cuuduongthancong.com].pdf
giai-tich-2__kha-vi-va-vi-phan - [cuuduongthancong.com].pdfgiai-tich-2__kha-vi-va-vi-phan - [cuuduongthancong.com].pdf
giai-tich-2__kha-vi-va-vi-phan - [cuuduongthancong.com].pdfVnNguyn914577
 
Ôn thi THPT Quốc Gia môn Toán về nguyên hàm và tích phân
Ôn thi THPT Quốc Gia môn Toán về nguyên hàm và tích phânÔn thi THPT Quốc Gia môn Toán về nguyên hàm và tích phân
Ôn thi THPT Quốc Gia môn Toán về nguyên hàm và tích phânLinh Nguyễn
 
Luận văn: Điểm bất động của một số lớp ánh xạ đa trị, HAY
Luận văn: Điểm bất động của một số lớp ánh xạ đa trị, HAYLuận văn: Điểm bất động của một số lớp ánh xạ đa trị, HAY
Luận văn: Điểm bất động của một số lớp ánh xạ đa trị, HAYViết thuê trọn gói ZALO 0934573149
 
Phongmath pp khu dang vo dinh
Phongmath   pp khu dang vo dinhPhongmath   pp khu dang vo dinh
Phongmath pp khu dang vo dinhphongmathbmt
 
Tổng hợp công thức giải nhanh trắc nghiệm toán THPT Quốc gia 2018
Tổng hợp công thức giải nhanh trắc nghiệm toán THPT Quốc gia 2018Tổng hợp công thức giải nhanh trắc nghiệm toán THPT Quốc gia 2018
Tổng hợp công thức giải nhanh trắc nghiệm toán THPT Quốc gia 2018Maloda
 
Phương pháp số và lập trình - Nội suy, Đạo hàm, Tích phân
Phương pháp số và lập trình - Nội suy, Đạo hàm, Tích phânPhương pháp số và lập trình - Nội suy, Đạo hàm, Tích phân
Phương pháp số và lập trình - Nội suy, Đạo hàm, Tích phânHajunior9x
 

Similar to Chuong 4 x (20)

Tổng hợp bồi dưỡng học sinh giỏi
Tổng hợp bồi dưỡng học sinh giỏiTổng hợp bồi dưỡng học sinh giỏi
Tổng hợp bồi dưỡng học sinh giỏi
 
Cac phuong phap giai pt ham thuong dung
Cac phuong phap giai pt ham thuong dungCac phuong phap giai pt ham thuong dung
Cac phuong phap giai pt ham thuong dung
 
Chuyen de hsg
Chuyen de hsgChuyen de hsg
Chuyen de hsg
 
Luận văn: Bổ đề đạo hàm logarit và ứng dụng, HAY, 9đ
Luận văn: Bổ đề đạo hàm logarit và ứng dụng, HAY, 9đLuận văn: Bổ đề đạo hàm logarit và ứng dụng, HAY, 9đ
Luận văn: Bổ đề đạo hàm logarit và ứng dụng, HAY, 9đ
 
06 mat101 bai2_v2.3013101225
06 mat101 bai2_v2.301310122506 mat101 bai2_v2.3013101225
06 mat101 bai2_v2.3013101225
 
Luận văn: Vài vấn đề cơ bản của hàm nhiều biến phức, HAY
Luận văn: Vài vấn đề cơ bản của hàm nhiều biến phức, HAYLuận văn: Vài vấn đề cơ bản của hàm nhiều biến phức, HAY
Luận văn: Vài vấn đề cơ bản của hàm nhiều biến phức, HAY
 
kỹ thuật giải phương trình hàm
kỹ thuật giải phương trình hàmkỹ thuật giải phương trình hàm
kỹ thuật giải phương trình hàm
 
13 ki-thuat-giai-phuong-trinh-ham (1)
13 ki-thuat-giai-phuong-trinh-ham (1)13 ki-thuat-giai-phuong-trinh-ham (1)
13 ki-thuat-giai-phuong-trinh-ham (1)
 
Luận văn: Lớp bài toán tìm giá trị lớn nhất giá trị nhỏ nhất, HOT
Luận văn: Lớp bài toán tìm giá trị lớn nhất giá trị nhỏ nhất, HOTLuận văn: Lớp bài toán tìm giá trị lớn nhất giá trị nhỏ nhất, HOT
Luận văn: Lớp bài toán tìm giá trị lớn nhất giá trị nhỏ nhất, HOT
 
Chứng minh bổ đề lagrange.doc
Chứng minh bổ đề lagrange.docChứng minh bổ đề lagrange.doc
Chứng minh bổ đề lagrange.doc
 
[Math educare] giao trinh toan cao cap a1-giai tich ham mot bien_chuoi so
[Math educare] giao trinh toan cao cap a1-giai tich ham mot bien_chuoi so[Math educare] giao trinh toan cao cap a1-giai tich ham mot bien_chuoi so
[Math educare] giao trinh toan cao cap a1-giai tich ham mot bien_chuoi so
 
Tom tat-li-thuyet-va-cong-thuc-giai-nhanh-toan-12-tran-quoc-nghia
Tom tat-li-thuyet-va-cong-thuc-giai-nhanh-toan-12-tran-quoc-nghiaTom tat-li-thuyet-va-cong-thuc-giai-nhanh-toan-12-tran-quoc-nghia
Tom tat-li-thuyet-va-cong-thuc-giai-nhanh-toan-12-tran-quoc-nghia
 
giai-tich-2__kha-vi-va-vi-phan - [cuuduongthancong.com].pdf
giai-tich-2__kha-vi-va-vi-phan - [cuuduongthancong.com].pdfgiai-tich-2__kha-vi-va-vi-phan - [cuuduongthancong.com].pdf
giai-tich-2__kha-vi-va-vi-phan - [cuuduongthancong.com].pdf
 
Ôn thi THPT Quốc Gia môn Toán về nguyên hàm và tích phân
Ôn thi THPT Quốc Gia môn Toán về nguyên hàm và tích phânÔn thi THPT Quốc Gia môn Toán về nguyên hàm và tích phân
Ôn thi THPT Quốc Gia môn Toán về nguyên hàm và tích phân
 
Luận văn: Điểm bất động của một số lớp ánh xạ đa trị, HAY
Luận văn: Điểm bất động của một số lớp ánh xạ đa trị, HAYLuận văn: Điểm bất động của một số lớp ánh xạ đa trị, HAY
Luận văn: Điểm bất động của một số lớp ánh xạ đa trị, HAY
 
Phongmath pp khu dang vo dinh
Phongmath   pp khu dang vo dinhPhongmath   pp khu dang vo dinh
Phongmath pp khu dang vo dinh
 
Tổng hợp công thức giải nhanh trắc nghiệm toán THPT Quốc gia 2018
Tổng hợp công thức giải nhanh trắc nghiệm toán THPT Quốc gia 2018Tổng hợp công thức giải nhanh trắc nghiệm toán THPT Quốc gia 2018
Tổng hợp công thức giải nhanh trắc nghiệm toán THPT Quốc gia 2018
 
CHUYÊN ĐỀ: LTĐH TOÁN 2014
CHUYÊN ĐỀ: LTĐH TOÁN 2014CHUYÊN ĐỀ: LTĐH TOÁN 2014
CHUYÊN ĐỀ: LTĐH TOÁN 2014
 
Phương pháp số và lập trình - Nội suy, Đạo hàm, Tích phân
Phương pháp số và lập trình - Nội suy, Đạo hàm, Tích phânPhương pháp số và lập trình - Nội suy, Đạo hàm, Tích phân
Phương pháp số và lập trình - Nội suy, Đạo hàm, Tích phân
 
Toan cao cap a1
Toan cao cap a1Toan cao cap a1
Toan cao cap a1
 

Chuong 4 x

  • 1. 1 1 Giải tích toán học. Tập 1. NXB Đại học quốc gia Hà Nội 2007. Từ khoá:Giải tích toán học, giải tích, Phép tích vi phân, Đạo hàm, vi phân, Công thức Taylor, Khai triển Maclaurin, Quy tắc L’hospital. Tài liệu trong Thư viện điện tử ĐH Khoa học Tự nhiên có thể được sử dụng cho mục đích học tập và nghiên cứu cá nhân. Nghiêm cấm mọi hình thức sao chép, in ấn phục vụ các mục đích khác nếu không được sự chấp thuận của nhà xuất bản và tác giả. Mục lục Chương 4 Phép tính vi phân của hàm một biến ....................................................................... 2 4.1 Đạo hàm và cách tính ....................................................................................................... 3 4.1.1 Định nghĩa đạo hàm................................................................................................... 3 4.1.2 Công thức đối với số gia của hàm số......................................................................... 3 4.2 Các qui tắc tính đạo hàm .................................................................................................. 4 4.2.1 Các qui tắc tính đạo hàm............................................................................................ 4 4.2.2 Đạo hàm của hàm số hợp........................................................................................... 4 4.2.3 Đạo hàm của hàm số ngược....................................................................................... 6 4.2.4 Đạo hàm theo tham số................................................................................................ 7 4.2.5 Đạo hàm một phía...................................................................................................... 7 4.2.6 Đạo hàm vô cùng ....................................................................................................... 9 4.2.7 Đạo hàm các hàm số sơ cấp....................................................................................... 9 4.3 Vi phân của hàm số ........................................................................................................ 10 4.3.1 Định nghĩa................................................................................................................ 10 Chương 4. Phép tính vi phân của hàm một biến Lê Văn Trực
  • 2. 2 4.3.2 Các qui tắc tính vi phân ........................................................................................... 11 4.3.3 Vi phân của hàm số hợp........................................................................................... 11 4.3.4 Ứng dụng của vi phân............................................................................................. 12 4.4 Các định lí cơ bản của hàm khả vi.................................................................................. 12 4.8.1 Cực trị địa phương ................................................................................................... 12 4.5 Đạo hàm và vi phân cấp cao........................................................................................... 18 4.8.1 Định nghĩa đạo hàm cấp cao.................................................................................... 18 4.8.2 Các công thức tổng quát đối với đạo hàm cấp n...................................................... 18 4.8.3 Vi phân cấp cao........................................................................................................ 19 4.6 Công thức Taylor............................................................................................................ 20 4.8.1 Công thức Taylor ..................................................................................................... 20 4.8.2 Khai triển Maclaurin................................................................................................ 22 4.7 Qui tắc L’hospital để khử dạng vô định ......................................................................... 25 4.8.1 Dạng vô định 0 0 ....................................................................................................... 25 4.8.2 Dạng vô dịnh ∞ ∞ ...................................................................................................... 27 4.8 Khảo sát hàm số.............................................................................................................. 30 4.8.1 Khảo sát đường cong cho dưới dạng phương trình hiện.......................................... 30 4.8.2 Đường cong cho dưới dạng tham số........................................................................ 32 4.8.3 Khảo sát đường cong trong tọa độ cực .................................................................... 36 4.9 Bài tập chương 4............................................................................................................. 39 Chương 4
  • 3. 3 3 Phép tính vi phân của hàm một biến 4.1 Đạo hàm và cách tính 4.1.1 Định nghĩa đạo hàm Giả sử U là một tập mở trong , :f U → và 0x U∈ . Cho x0 một số gia 0xΔ ≠ đủ nhỏ sao cho 0x x U+ Δ ∈ . Khi đó ta gọi 0 0( ) ( )y f x x f xΔ = + Δ − là một số gia của hàm số tương ứng với số gia đối số xΔ tại điểm x0. Xét tỷ số giữa số gia hàm số với số gia đối số. Nếu tỷ số dẫn đến một giới hạn hữu hạn xác định khi 0xΔ → , thì ta nói rằng hàm f khả vi tại điểm x0, giới hạn đó gọi là đạo hàm của hàm số tại x0 và ký hiệu là 0 0 0 0 ( ) ( ) ( ) lim x f x x f x f x xΔ → + Δ − ′ = Δ . (4.1.1) Các ký hiệu y′ hay ( )f x′ là các ký hiệu đạo hàm theo Largrange, còn dy dx hay 0( )df x dx là các kí hiệu theo Leibnitz và Dy hay Df(x0) là các kí hiệu theo Cauchy. Đôi khi để nhấn mạnh biến số lấy đạo hàm, người ta thường viết biến đó thành chỉ số dưới: 0 0, ( ), hay ( )′ ′x x x xy f x D y D f x (4.1.2) Hàm f được gọi là khả vi trên U nếu nó khả vi tại mọi điểm thuộc U. 4.1.2 Công thức đối với số gia của hàm số Nếu hàm y = f(x) khả vi tại 0 ,∈x U ta có thể biểu diễn số gia của hàm số 0 0 0( ) ( ) ( )Δ = Δ = + Δ −y f x f x x f x như sau. Theo định nghĩa 0 0 0 ( ) lim ( ) Δ → Δ ′= Δx f x f x x . Đặt 0 0 ( ) ( ) α Δ ′= + Δ f x f x x với 0α → khi 0Δ →x . (4.1.3) Ta có 0 0( ) ( ) .α′Δ = Δ + Δf x f x x x với 0 lim 0α Δ → → x . (4.1.4) Kí hiệu . ( )α Δ = ο Δx x và hiển nhiên 0 ( ) lim 0 Δ → ο Δ = Δx x x . Do đó (4.1.4) có thể viết dưới dạng 0 0( ) ( ) ( ).′Δ = Δ + ο Δf x f x x x (4.1.5) Định lý 4.1.1 Nếu hàm y = f(x) khả vi tại 0x U∈ thì f(x) liên tục tại x0. Chứng minh: Thật vậy ta có
  • 4. 4 0 0 0( ) ( ) ( ) ( )′+ Δ − = Δ + ο Δf x x f x f x x x , suy ra [ ]0 0 0 0 0 0 0 0 0 lim ( ) ( ) lim ( ) lim ( ) lim ( ) ( ). Δ → Δ → Δ → Δ → ′+ Δ − = Δ + ο Δ ⇒ + Δ = x x x x f x x f x f x x x f x x f x 4.2 Các qui tắc tính đạo hàm 4.2.1 Các qui tắc tính đạo hàm Trước hết ta hãy nhắc lại các qui tắc tính đạo hàm đã biết Định lí 4.2.1 Cho , :f g U → , trong đó U là tập hợp mở trong R, còn f, g là hai hàm khả vi tại 0x U∈ . Khi đó 1 2,c c∀ ∈ các hàm 1 2 ,c f c g+ .f g và f g (nếu g(x0) 0≠ cũng là các hàm khả vi tại điểm x0 và ta có các công thức sau: a) 1 2 0 1 0 2 0( ) ( ) ( ) ( )c f c g x c f x c g x′ ′ ′+ = + (4.2.1) b) 0 0 0 0 0( , ) ( ) ( ) ( ) ( ). ( )f g x f x g x g x f x′ ′ ′= + (4.2.2) c) 0 0 0 0 0 02 0 0 ( ) ( ) ( ). ( ) ( ) , ( ) ( ) f x g x g x f xf x g x g g x ′ ′ ′−⎛ ⎞ = ≠⎜ ⎟ ⎝ ⎠ . (4.2.3) 4.2.2 Đạo hàm của hàm số hợp Định lí 4.2.2 Cho :g U V→ và :f V → trong đó U, V là hai tập hợp mở trong , hàm u=g(x) khả vi tại 0x U∈ và hàm y=f(u) khả vi tại u0=g(x0) V∈ . Khi đó hàm hợp 0f g khả vi tại x0 và ta có công thức 0 0 0 0( ) ( ) ( ( )) ( )f g x f g x g x′ ′ ′= (4.2.4) hay gọn hơn .x u xy y u′ ′ ′= . (4.2.5) Chứng minh: Theo công thức (4.1.5) hàm f khả vi tại u0, nên ta có 0 0 0( ) ( ) ( ) ( )uf f u u f u f u u u′Δ = + Δ − = Δ + ο Δ . Mặt khác hàm g khả vi tại x0 nên 0 0 0( ) ( ) ( ) ( )xu g x x g x g x x x′Δ = + Δ − = Δ + ο Δ . Thế uΔ vào biểu thức fΔ ta được [ ]0 0 0 0 0 0 0 ( ) ( ) ( ) ( ) ( ) ( ) = ( ). ( ) ( ) ( ) ( ). u x u x u f u u f u f u g x x x u f u g x x f u x u ο ο ′ ′+ Δ − = Δ + Δ + ο Δ ′ ′ ′Δ + Δ + ο Δ Chia cả 2 vế cho xΔ
  • 5. 5 5 0 0 0 0 0 ( ) ( ) ( ) ( ) ( ). ( ) ( ) .u x u f u u f u x u f u g x f u x x x + Δ − ο Δ ο Δ ′ ′ ′= + + Δ Δ Δ Ta thấy do hàm u liên tục tại x0 nên khi 0xΔ → thì 0uΔ → và 0 0 0 0 ( ) ( ( )) ( ), ( ) ( ) ( ( )) ( ). o o f u f g x f g x f u u f u f g x f g x = = + Δ = = = Bây giờ ta hãy viết lại biểu thức trên dưới dạng: 0 0 0 0 0 0 ( ) ( ) ( ) ( ) ( ). ( ) ( ) . .u x u f g x f g x x u u f u g x f u x x u x − ο Δ ο Δ Δ ′ ′ ′= + + Δ Δ Δ Δ Cho 0xΔ → ta được 0 0 0 0( ) ( ) ( ( )). ( ),uf g x f g x g x′ ′ ′= và công thức được chứng minh. Ví dụ 3: i) Ta thấy 0lnx x a a e a= ∀ > nên ln ( ) ( )x x a a e′ ′= , đặt u = xlna, ln ( )' .ln lnu x a x e e a a a= = Do đó ta có công thức sau ln( )x x a a′ = a với 0a∀ > . (4.2.6) ii) Ta có 0ln x e xα α = ∀ >x và α∀ ∈ Do đó: 1 1ln ln ( ) ( ) . . . .x x x e e x x x α α α α α α′ ′= = = . Và ta có công thức sau: 1 ( ) .x xα α α − ′ = . (4.2.7) Ví dụ 4: Tính 1 1 cos x x d I e dx − + = với 1x ≠ − Đặt 1 1 cos x u x − = + 1 1 1 1 cos . . cos x u u x x d x I e e u e dx x − + ′−⎛ ⎞ ′= = = ⎜ ⎟+⎝ ⎠ Lại đặt 1 1 x v x − = + ta có 2 1 1 1 2 1 1 1 1 (cos ) sin . sin sin . ( ) x x x v v v x x x x ′− − −⎛ ⎞ ′ ′= − = − = −⎜ ⎟+ + + +⎝ ⎠ Cuối cùng 1 1 2 1 1 2 11 cos . .sin ( ) x x x I e xx − + −⎛ ⎞ = − ⎜ ⎟++ ⎝ ⎠ . Ví dụ 5: Cho , :f g U → trong đó f(x)>0, x U∀ ∈ và tồn tại ( ), ( )f x g x′ ′ với x U∈ .
  • 6. 6 Khi đó ( ) ( )ln ( ) ( )ln ( ) ( ) ( ( )) ( ( ).ln ( )) ( ) =( ( )) . ( )ln ( ) ( ). . ( ) g x g x f x g x f x g x d d d f x e e g x f x dx dx dx f x f x g x f x g x f x ⎡ ⎤ ⎡ ⎤= =⎣ ⎦ ⎣ ⎦ ′⎡ ⎤ ′ +⎢ ⎥ ⎣ ⎦ 4.2.3 Đạo hàm của hàm số ngược Định lí 4.2.4 Giả sử hàm f(x) khả vi liên tục trên (a,b) với 0( )f x′ ≠ ( , )x a b∀ ∈ . Khi đó hàm f(x) đơn điệu thực sự nên có hàm ngược x = g(y), : ( ( ), ( )) ( , ).g f a f b a b→ Khi đó g(y) cũng khả vi tại y = f(x) và có đạo hàm g’(y) thoả mãn hệ thức: 1 ( ) ( ) g y f x ′ = ′ (4.2.8) hay gọn hơn: 1 y x x y ′ = ′ . (4.2.9) Chứng minh: Do (g.f)(x) = x ( , )x a b∀ ∈ Hay ( ( ))g f x x= ( , )x a b∀ ∈ . Lấy đạo hàm hai vế đẳng thức trên theo x ta được 1 hay 1( ( )). ( ) ( ). ( )g f x f x g y f x′ ′ ′ ′= = suy ra 1 ( ) ( ) g y f x ′ = ′ ( , )x a b∀ ∈ . Ví dụ 6: i) Xét hàm số y = arcsinx với −1< x <1 và 2 2 y π π − < < . Ta biết rằng y = arcsinx, tương đương với x = siny, do đó do 2 2 cos , ,yx y y π π⎛ ⎞ ′ = ∈ −⎜ ⎟ ⎝ ⎠ thì 0cos y > nên 2 1yx x′ = − , suy ra 2 1 1 xy x ′ = − . Tương tự, tao có các công thức sau: ii) y = arccosx với −1< x <1, 2 1 1 xy x ′ = − − iii) y = arctgx với x−∞ < < +∞ , 2 1 1 xy x ′ = +
  • 7. 7 7 iv) y = arccotgx với x−∞ < < +∞ , 2 1 1 xy x − ′ = + . 4.2.4 Đạo hàm theo tham số Xét hàm y của biến x được cho dưới dạng tham số ( ) ( ) x x t y y t =⎧ ⎨ =⎩ với ( , ).t α β∈ Giả sử x là hàm khả vi, liên tục và '( ) 0x t ≠ ( , )t α β∈ . Khi đó x(t) là hàm đơn điệu thực sự trên ( , )α β , vì vậy nó có hàm ngược t = t(x). Khi đó ta có hàm hợp y = y(t) = y(t(x)). Hãy tính xy′ . Cho t một số gia Δt, Δx là số gia tương ứng của Δt, Δy là số gia tương ứng của Δx. Ta có y y t xx t Δ Δ Δ= ΔΔ Δ suy ra 0 0 0 lim lim lim t t x x t t y yy ty xx x t Δ → Δ → Δ → Δ ′Δ Δ′ = = = Δ ′Δ Δ . (4.2.10) Ví dụ 7: Xét hàm số 1( sin ), ( cos )x a t t y a t= − = − với 0 2( , )t π∈ . Khi đó 2 2 2 2 1 22 2 sin cos sin ( ) cot g ( cos ) sin t t a t t y x ta t ′ = = = − . 4.2.5 Đạo hàm một phía Giả sử f(x) được xác định trên (a,b) và 0 ( , )x a b∈ . Ta nói giới hạn hữu hạn, nếu tồn tại 0 0 0 0 ( ) ( ) lim lim x x f x x f xy x x+ + Δ → Δ → + Δ −Δ = Δ Δ (4.2.11) là đạo hàm bên phải của hàm f(x) tại điểm x0, kí hiệu là 0( )+ ′f x (xem hình 4.2.1). Tương tự, ta có đạo hàm bên trái của hàm f(x) tại điểm x0 kí hiệu là 0( ):− ′f x 0 0 0 0 0 ( ) ( ) lim lim ( ) x x f x x f xy f x x x− − − Δ → Δ → + Δ −Δ ′= = Δ Δ (4.2.12) Ta thấy muốn có 0( )f x A′ = điều kiện cần và đủ là 0 0( ) ( )f x f x A+ − ′ ′= = .
  • 8. 8 Hình 4.2.1 Ví dụ 8: Cho hàm f(x) =|x|, hãy xét đạo hàm của hàm số tại x0 = 0. Ta có 0 0y f x f xΔ = + Δ − = Δ( ) ( ) | | , 0 0 0 0 0 1 0 1 ( ) lim lim , ( ) lim lim . x x x x y x f x x y x f x x + + − − + Δ → Δ → − Δ → Δ → Δ Δ ′ = = = Δ Δ Δ −Δ ′ = = = − Δ Δ Vậy hàm f(x) liên tục tại x0 = 0, nhưng f’(0) không tồn tại. Ví dụ 9: Cho hàm số 3 khi 0 khi 0 sin ( ) x x f x x a x ⎧ ≠⎪ = ⎨ ⎪ =⎩ 1) Tìm a để hàm số liên tục tại x = 0. 2) Với a tìm được, hãy xét sự khả vi của hàm số tại x = 0 Giải: 1) Do 3 2 0 0 0 sin sin lim lim sin x x x x x x x→ → = = Vậy để hàm liên tục tại x = 0 thì phải có a = 0. 2) Với a=0 ta có 3 khi 0 khi 0 sin ( ) 0 x x f x x x ⎧ ≠⎪ = ⎨ ⎪ =⎩ Ta thấy 3 0 0 0 0 0 ( ) ( ) sin lim lim x x f x f x x x→ → − = = − . Vậy 0 0( )f ′ = và hàm khả vi tại x=0. Ví dụ 10: Chứng minh rằng hàm số f(x) =|x−a| ( )ϕ x , trong đó ( )ϕ x là hàm liên tục và 0( )aϕ ≠ , không khả vi tại x = a.
  • 9. 9 9 Ta có 0 0 ( ) ( ) | | ( ) ( ) lim lim x x f a x f a x a x f x x x ϕ → → + Δ − Δ + Δ ′ = = Δ Δ . Suy ra: ( )+ ′f a = ( )ϕ a và ( )− ′f a =– ( )ϕ a . Do ( ) ( )+ − ′ ′≠f a f a nên hàm số f(x) không khả vi tại x=a. 4.2.6 Đạo hàm vô cùng Nếu 0 0 0 0 hay ( ) ( ) lim lim x x f x x f xy x xΔ → Δ → + Δ −Δ = = +∞ − ∞ Δ Δ thì ta nói rằng tại x = x0 hàm f(x) có đạo hàm vô cùng. Khi đó tiếp tuyến với đồ thị f(x) tại x = x0 song song với trục Oy. Ta cần chú ý rằng nếu như 0( )f x′ không là hữu hạn thì hàm f(x) không nhất thiết phải liên tục tại điểm x0. Ví dụ xét hàm 1 khi 0 0 khi 0 1 khi 0 ( ) . x f x x x − <⎧ ⎪ = =⎨ ⎪ >⎩ Với 0xΔ ≠ , ta có 0 1( ) ( ) | | f x f x x Δ − = Δ Δ , do đó 0( )f ′ = +∞ nhưng đương nhiên f(x) không liên tục tại điểm x0 = 0. 4.2.7 Đạo hàm các hàm số sơ cấp Sau đây là bảng đạo hàm của một số hàm sơ cấp: 1 0 2 1 ) ) y c y y x y ′= = ′= = 1 2 3 1 1 1 1 2 ) , , .y x R y x y y x x y x y x α α α α α − ′= ∈ ≠ − = − ′= = ′= = 4) x x y e y e′= = x y a= với 0 lnx a y a a′> = 5) logay x= với 1 0 ln a y x a ′> = 1 lny x y x ′= = 6) sin cosy x y x′= =
  • 10. 10 7) cos siny x y x′= = − 2 2 1 8 tg) sec cos y x y x x ′= = = 2 2 1 9) cot g cosec sin y x y x x ′= = − = − 2 1 10) arcsin 1 y x y x ′= = − 2 1 11) arccos 1 y x y x ′= = − − 2 1 12) arctg 1 y x y x ′= = + 2 1 13) arccot g 1 y x y x ′= = − + 14) sh chy x y x′= = 15) ch shy x y x′= = 2 1 16) th ch y x y x ′= = 2 1 17) cth sh y x y x − ′= = 2 1 18) argsh 1 y x y x ′= = + 2 1 19) arg ch 1 y x y x ′= = − 2 1 20) arg th 1 y x y x ′= = − 2 1 21) arg cth . 1 y x y x ′= = − 4.3 Vi phân của hàm số 4.3.1 Định nghĩa Cho hàm y = f(x) xác định trên tập hợp mở U ⊂ và 0x U∈ . Cho x0 một số gia 0xΔ ≠ đủ nhỏ sao cho 0x x U+ Δ ∈ . Giả sử f(x)khả vi tại 0x U∈ , khi đó 0 0 0( ) ( ) ( ) ( )f x x f x f x x xο′+ Δ − = Δ + Δ . (4.3.1)
  • 11. 11 11 Ta gọi biểu thức 0( )f x x′ Δ là vi phân của hàm f(x) tại điểm x0 ứng với số gia xΔ của đối số và kí hiệu là 0 0( , ) ( )df x x f x x′Δ = Δ . (4.3.2) Bây giờ ta xét trường hợp đặc biệt khi f(x) = x. Ta có ( ) 1f x′ = , do đó dx = 1. xΔ = xΔ , vì thế trong biểu thức (4.3.2) ta có thể viết dx thay cho xΔ và dx gọi là vi phân của biến số độc lập. Từ đây, ta có thể xác định vi phân của hàm f tại x U∈ theo công thức Df = ( )f x′ dx (4.3.3) hay dy = ( )y x′ dx (4.3.4) Hệ thức này giải thích lí do ta kí hiệu đạo hàm của hàm y = f(x) là ( ) dy y x dx ′ = . 4.3.2 Các qui tắc tính vi phân Từ các qui tắc tính đạo hàm, ta dễ dàng suy ra các qui tắc tương ứng cho vi phân. 1 2 1 2 1 2) ( ) ,i d c f c g c df c dg c c+ = + ∀ ∈ ) ( . )ii d f g gdf fdg= + (4.3.5) 2 ) ( ) nÕu 0 f gdf fdg iii d g g g − = ≠ . 4.3.3 Vi phân của hàm số hợp Giả sử các hàm y = f(x) và x = g(t) sao cho đối với chúng có thể thiết lập hàm hợp y = f(g(t)). Nếu tồn tại các đạo hàm xy′ và tx′ thì theo quy tắc đạo hàm hàm hợp sẽ tồn tại đạo hàm ty′ = xy′ . tx′ . (4.3.6) Nếu xem x là biến độc lập thì vi phân dy được biểu thị bởi công thức (4.3.4). Bây giờ ta xem x là hàm của biến t, ta có tdy y dt′= (4.3.7) Tuy nhiên nếu thay đạo hàm ty′ bởi biểu thức (4.3.6) và chú ý rằng dx = tx′dt, thì cuối cùng ta được x t xdy y x dt y dx′ ′ ′= = hay dy = ( )y x′ dx, tức là quay trở lại dạng ban đầu của vi phân. Như vậy, ta luôn luôn có quyền viết vi phân của y dưới dạng (4.3.4) dù x có phải là biến độc lập hay không . Điều khác nhau chỉ là ở chỗ, nếu chọn t là biến độc lập thì dx không phải là số gia tuỳ ý mà là vi phân của x xem là hàm của t. Tính chất đó gọi là tính bất biến của dạng vi phân, Ví dụ 1: Cho hàm số 1 ln 1 x x e y e + = − , hãy tính dy
  • 12. 12 Ta thấy 2 2 1 1 2 2 . 1 1 1 1 x x x x x x x x e e e e y dy dx e e e e ′⎛ ⎞− + − ′ = = ⇒ = −⎜ ⎟ + − − −⎝ ⎠ Ví dụ 2: Tính: (sin ) (cos ) d x d x Ta có: (sin ) cos cot g (cos ) sin d x xdx x d x xdx = = − − với , .x k kπ≠ ∈ 4.3.4 Ứng dụng của vi phân Cho hàm y = f(x) xác định trên tập mở U ⊂ và 0x U∈ . Giả sử f khả vi tại 0x U∈ . Cho x0 một số gia h sao cho 0x h U+ ∈ , khi đó 0 0 0 0( , ) ( ) ( ) ( ) ( )f x h f x h f x f x h hο′Δ = + − = + . (4.3.8) Nếu |h| đủ nhở thì ( )hο nhỏ tuỳ ý và ta có xấp xỉ 0 0 0( ) ( ) ( )f x h f x f x h′+ − ≈ hay 0 0 0( ) ( ) ( )f x h f x f x h′+ ≈ + . (4.3.9) Ví dụ 3: Tính gần đúng arctg1,05 . Theo công thức (4.3.9), ta có 12 1 arctg1,05 arctg1 | .0,05 0,81 1 x x =≈ + ≈ + . Ví dụ 4: Tính gần đúng arcsin 0,05 Theo công thức (4.3.9), ta có 0 2 1 arcsin 0,05 arcsin 0 | .0,05 0.05 1 x x =≈ + = − . 4.4 Các định lí cơ bản của hàm khả vi 4.8.1 Cực trị địa phương Cho hàm f(x) xác định trên khoảng (a,b). Ta nói rằng hàm f(x) đạt cực đại địa phương tại điểm ( , )c a b∈ nếu tồn tại một số 0δ > sao cho ( ) ( ) ( , ).f x f c x c cδ δ≤ ∀ ∈ − + (4.4.1) Hàm f đạt cực tiểu địa phương tại ( , )c a b∈ nếu: ( ) ( ) ( , )f x f c x c cδ δ≥ ∀ ∈ − + . (4.4.2) Điểm mà tại đó hàm đạt cực đại hoặc cực tiểu địa phương gọi chung là điểm cực trị.
  • 13. 13 13 Định lí Ferma Cho : ( , )f a b → , nếu hàm đạt cực trị tại ( , )c a b∈ và nếu f(x) khả vi tại c thì ( ) 0f c′ = . (4.4.3) Chứng minh: Giả sử hàm đạt cực đại tại c (trường hợp đạt cực tiểu tại c chứng minh tương tự). Do hàm đạt cực đại tại c nên ∀h đủ nhỏ ta có 0( ) ( )f c h f c h+ − ≤ ∀ suy ra 0 0 ( ) ( )f c h f c h h + − ≤ ∀ > 0 0 ( ) ( )f c h f c h h + − ≥ ∀ < . Cho nên 0 0 ( ) ( ) ( ) lim h f c h f c f c h++ → + − ′ = ≤ và 0 0 ( ) ( ) ( ) lim h f c h f c f c h−− → + − ′ = ≥ . Mặt khác vì f có đạo hàm tại điểm c nên ( ) ( ) ( )f c f c f c+ − ′ ′ ′= = , do đó 0( )f c′ = (xem hình 4.4.1) Hình 4.4.1 Chú ý rằng sự triệt tiêu của đạo hàm ( )′f c về phương diện hình học có ý nghĩa là tiếp tuyến tại điểm tương ứng của đường cong song song với trục Ox. Định lý Rolle Cho hàm :[ , ]f a b → có tính chất sau: i) f(x) liên tục trên [a,b], ii) f(x) khả vi trên (a,b), iii) f(a)=f(b). Khi đó tồn tại ít nhất một điểm ( , )c a b∈ sao cho ( ) 0f c′ = . Chứng minh: Do f(x) liên tục trên đoạn [a,b] nên theo định lí Weierstrass thứ hai hàm f(x) sẽ đạt giá trị lớn nhất M và giá trị bé nhất m trên đoạn [a,b]: [ , ] [ , ] max ( ), min ( ). x a b x a b M f x m f x ∈ ∈ = = Ta hãy xét hai khả năng có thể xảy ra:
  • 14. 14 1) M = m. Khi đó từ bất đẳng thức ( ) [ , ]m f x M x a b≤ ≤ ∀ ∈ suy ra ( ) , [ , ]f x m x a b= ∀ ∈ Vì vậy 0( ) , [ , ]f x x a b′ = ∀ ∈ . Do đó điểm c là lấy điểm bất kì thuộc khoảng (a,b). 2) m<M. Do f(a)=f(b), hàm f(x) không thể đạt cả hai giá trị m, M tại hai đầu mút của khoảng, có nghĩa là ít nhất một trong hai giá trị đó đạt tại một điểm ( , )c a b∈ . Khi đó, theo định lí Fermat ( ) 0f c′ = . Định lí đã được chứng minh. Chú ý: Ta chú ý rằng giả thiết f(x) liên tục trên [a,b] là một giả thiết không thể bỏ qua được. Ví dụ như xét hàm số (xem hình 4.4.2) khi 0 1 1 khi 0 ( ) x x f x x < ≤⎧ = ⎨ =⎩ Cho dù f(0)=f(1), nhưng hàm số không liên tục trên [0,1], nên không thể áp dụng định lí Rolle được (đạo hàm không nơi nào bằng 0 trên (0,1)). Giả thiết hàm f(x) khả vi trong khoảng (a,b) cũng là một giả thiết không thể bỏ qua được. Chẳng hạn xét hàm số (xem hình 4.4.3) 1 2 1 2 Hình 4.4.2 Hình 4.4.3 1 ví i 0 2( ) 1 1 ví i 1 2 x x f x x x ⎧ ≤ ≤⎪⎪ = ⎨ ⎪ − < ≤ ⎪⎩ Hàm số này liên tục trên đoạn [0,1], f(0)=f(1), nhưng không có đạo hàm tại 1 2 x = , do đó cũng không áp dụng định lí Rolle được. Ví dụ 1: Hàm số f(x)=1− 23 x triệt tiêu khi x1=−1, x2=1 nhưng ′ ≠( ) 0f x với |x|≤ 1 . Điều này không mâu thuẫn với định lí Rolle. Định lí về số gia hữu hạn (Định lí Lagrange). Giả sử :[ , ]f a b → có các tính chất i) f liên tục trên [a,b]
  • 15. 15 15 ii) f khả vi trên (a,b) Khi đó tồn tại ít nhất một điểm ( , )c a b∈ sao cho: ( ) ( ) ( ) f b f a f c b a − ′= − . (4.4.4) Chứng minh: Ta hãy xét hàm bổ trợ sau: ( ) ( ) ( ) ( ) ( ) ( ) f b f a F x f x f a x a b a − = − − − − . Hiển nhiên F(x) liên tục trên [a,b] vì nó là hiệu của hàm liên tục f(x) và hàm tuyến tính. Trong khoảng (a,b) hàm đó có đạo hàm hữu hạn bằng: ( ) ( ) ( ) ( ) . f b f a F x f x b a − ′ ′= − − y xbca0 C B A f(b) f(a) Hình 4.4.4 Cuối cùng ta thấy F(a)=F(b)=0. Theo định lí Rolle tồn tại một điểm ( , )c a b∈ sao cho ( ) 0F c′ = . Như vậy ( ) ( ) ( ) 0 f b f a f c b a − ′ − = − . Do đó ( ) ( ) ( ) f b f a f c b a − ′ = − Ý nghĩa hình học: Tỷ số ( ) ( )f b f a b a − − là hệ số góc của cát tuyến AB, còn ( )f c′ là hệ số góc của tiếp tuyến với đường cong y=f(x) tại điểm C(c,f(c)). Theo định lí Lagrange trên cung AB tìm được ít nhất một điểm c, mà tại đó tiếp tuyến song song với dây cung AB. Trường hợp f(a) = f(b) ta có định lí Rolle. Chú ý 1: Bởi vì ( , )c a b∈ , nên ta có thể viết c = a + ( ), 0 1b aθ θ− < < . Khi đó công thức Lagrange có thể viết dưới dạng
  • 16. 16 f(b) − f(a )= [ ( )]( ),0 1f a b a b aθ θ+ − − < < . (4.4.5) Chú ý 2: Nếu đặt a = x, b = x+ xΔ thì ta nhận được f(x + Δx) − f(x)=f’(x + Δθ x)Δx trong đó θ< <0 1 (4.4.6) Định lí Cauchy Giả thiết i) Các hàm f(x) và g(x) xác định và liên tục trên[a,b] ii) f(x) và g(x) khả vi trên (a,b) iii) ′ ≠ ∀ ∈( ) 0 ( , ).g x x a b Khi đó tồn tại ít nhất một điểm ( , )c a b∈ sao cho ( ) ( ) ( ) ( ) ( ) ( ) f b f a f c g b g a g c ′− = ′− . (4.4.7) Rõ ràng rằng định lí Lagrange là trường hợp đặc biệt của định lí Cauchy: Để được công thức số gia hữu hạn thì trong công thức Cauchy (4.4.5) ta đặt g(x)=x. Chứng minh: Trước hết ta để ý rằng theo định lí Lagrange ta có thể tìm được một số 1 ( , )c a b∈ sao cho: 1( ) ( ) ( )( )g b g a g c b a′− = − Theo giả thiết 1( ) 0g c′ ≠ , nên ( ) ( ) 0.g b g α− ≠ (4.4.8) Bây giờ ta xét hàm số ( ) ( ) ( ) ( ) ( ) ( ( ) ( )) ( ) ( ) f b f a F x f x f a g x g a g b g a − = − − − − . (4.4.9) Ta thấy hàm số thoả mãn tất cả các giả thiết của định lí Rolle. Thật vậy F(x) liên tục, đạo hàm ( )F x′ tồn tại trong khoảng (a,b), cụ thể bằng ( ) ( ) ( ) '( ) ( ) ( ) ( ) f b f a F x f x g x g b g a − ′ ′= − − , và hiển nhiên F(a)=F(b). Do đó theo định lí Rolle ( , )c a b∃ ∈ sao cho ( ) 0.F c′ = Nói cách khác ( ) ( ) ( ) ( ) 0 ( ) ( ) f b f a f c g c g b g a − ′ ′− = − . Từ đây suy ra định lí được chứng minh. Ví dụ 1: Cho hàm số 2 3 khi 0 1 2( ) 1 khi 1 . x x f x x x ⎧⎪ −⎪ ≤ ≤⎪⎪⎪= ⎨ ⎪⎪ < < +∞⎪⎪⎪⎩ Xác định giá trị trung gian c của công thức số gia hữu hạn đối với hàm số ( )f x trên đoạn [0,2]. Trước hết ta thấy
  • 17. 17 17 1 1 1 1 ( ) (1) (1) lim lim 1 1 1x x f x f xf x x+ ++ → → − − ′ = = = − − − 2 1 1 3 1 ( ) (1) 2(1) lim lim 1 1 1x x x f x f f x x− −− → → − − − ′ = = = − − − . Vậy theo định nghĩa hàm số f(x) có đạo hàm tại x=1 và (1) 1f ′ = − . Do đó ta có 2 2 khi 0 1 khi 0 1 ( ) 1 khi 1 1 khi 1< 2 1 khi 1 2 x x x x f x x x x x x ⎧ ⎪− ≤ < − ≤ ≤⎧ ⎪ ⎪ = − = = −⎨ ⎨ ≤⎪ ⎪− ⎩⎪ < ≤ ⎩ Công thức số gia hữu hạn đối với hàm số f(x) trên [0,2] là (2) (0) ( )(2 0)f f f c′− = − hay 1 3 2. ( ) 2 2 f c′− = , suy ra: 1 ( ) 2 f c − ′ = . Mặt khác theo biểu thức đạo hàm ( )f x′ , ta có Khi 0< c <1, 1 2 c− = − suy ra 1 2 c = , Khi 1< c <2, 2 1 1 , 2c − = − suy ra 2c= . Ví dụ 2: Giả sử f(x) khả vi trên đoạn [0,1] và (0). (1) 0f f′ ′ < . Chứng minh (0,1)ξ∃ ∈ sao cho ( ) 0f ξ′ = . Chứng minh: Thật vậy, theo giả thiết hàm số liên tục trên [0,1], nên đạt giá trị lớn nhất, bé nhất trên [0,1]. Không mất tổng quát, giả sử (0) 0, (1) 0f f+ − ′ ′< > , ta có 0 ( ) (0) (0) lim 0, x f x f f x++ → − ′ = < suy ra ( ) (0) 0 hay ( ) (0) 0 f x f f x f x − < − < với 0x > khá bé. Hơn nữa do 1 ( ) (1) (1) lim 0 1x f x f f x−− → − ′ = > − nên ( ) (1) 0 1 f x f x − > − hay ( ) (1)f x f− <0, hay ( ) (1)f x f< khi x khá gần 1, x<1. Từ lý luận trên suy ra giá trị bé nhất của hàm số trên [0,1] không thể xảy ra ở hai đầu mút 0 và 1. Vậy giá trị bé nhất đạt được tại (0,1)ξ ∈ . Theo định lý Ferma
  • 18. 18 ( ) 0f ξ′ = Ví dụ 3: Chứng minh nếu ( )xϕ là hàm khả vi, đơn điệu tăng và | ( )| ( )f x xϕ′ ′≤ khi 0x x≥ , thì ta có 0 0 0| ( ) ( )| ( ) ( ) khif x f x x x x xϕ ϕ− ≤ − ≥ . Chứng minh: Theo định lý Cauchy 0 0 ( ) ( ) ( ) 1 ( ) ( ) ( ) f x f x f c x x cϕ ϕ ϕ ′− = < ′− với 0x c x< < . Từ đây suy ra: 0 0 0| ( ) ( )| ( ) ( ) khif x f x x x x xϕ ϕ− ≤ − > . Cuối cùng chú ý rằng đẳng thức trên hiển nhiên đúng khi x = x0. 4.5 Đạo hàm và vi phân cấp cao 4.8.1 Định nghĩa đạo hàm cấp cao Giả sử :f U → là hàm khả vi trên tập mở U ⊂ , khi đó ta nhận được hàm :f U′ → . Nếu tại 0x U∈ , ( )f x′ có đạo hàm thì ta gọi đạo hàm của ( )f x′ tại 0x là đạo hàm cấp hai của hàm f(x) tại 0x và kí hiệu là 0( )f x′′ . Hàm f có đạo hàm cấp hai tại 0x còn gọi là khả vi cấp hai tại 0x . Một cách tổng quát, đạo hàm của đạo hàm cấp (n−1) được gọi là đạo hàm cấp n của hàm f(x) và kí hiệu là ( ) ( )( ) ( ); ; ; n n n n n n d f x d y f x y dx dx . Đương nhiên là ( ) ( ) ( ) ( ( )) ( )m n m n f x f x+ = . (4.5.1) Đôi khi ta viết f(0) thay cho f. Ta chú ý rằng, nếu tồn tại ( ) 0( )n f x , tức là nếu hàm ( 1) ( )n f x− có đạo hàm tại điểm 0x , thì hàm ( 1) ( )n f x− được xác định không chỉ tại 0x , mà là trong toàn bộ khoảng 0 0( , )x xδ δ− + , trong đó là δ số dương được chọn thích hợp. Trong khoảng này những hàm ( 2) ( 3) ( ); ( ),...,n n f x f x− − ( ), ( )f x f x′ được xác định. 4.8.2 Các công thức tổng quát đối với đạo hàm cấp n Giả sử :f U → và :g U → là hai hàm khả vi cấp n trên U. Khi đó 1 2 , .c f c f f g+ là những hàm khả vi cấp n trên U, trong đó 1 2,c c ∈ và ( ) ( ) ( ) 1 2 1 2) ( ) ( ) ( ) ( )n n n i c f c g x c f x c g x+ = + (4.5.2) ( ) ( ) ( ) 0 )( . ) ( ) ( ). ( ) n n k k n k n k ii f g x C f x g x− = = ∑ . (4.5.3)
  • 19. 19 19 Công thức (4.5.3) còn gọi là công thức Leibnitz. Ví dụ 1: Tính đạo hàm cấp 3 của hàm số 3 siny x x= Đặt 3 , sinf x g x= = . Khi đó (3) (3) (2) (2) (3) . 3 . 3 . .y f g f g f g f g′ ′= + + + hay (3) 3 2 sin 9 sin 18 cos 6sin .y x x x x x x x= − − + + Ví dụ 2: Tính đạo hàm cấp 3 của hàm số sau ( ) 3 ( )y xf x a f a x′= − + − trong đó a là hằng số. Đặt 3 , ( ), ( ), ( )u x v f x a v f a x v f a x′ ′ ′′ ′′= = − = − − = − , (3) (4) ( )v f a x= − − , khi đó (3) (3) (4) (3) (3) (4) 3 ( ) ( ) 3 ( ) hay ( ). y f a x xf a x f a x y xf a x = − − − − − = − − 4.8.3 Vi phân cấp cao Cho U mở trong và f là hàm khả vi cấp n trên tập mở U. Ta gọi vi phân cấp hai của hàm f, ký hiệu là d2 f là biểu thức d2 f=d(df). Một cách tổng quát, ta gọi vi phân cấp n của hàm f là vi phân của vi phân cấp n−1 của hàm f: 1 ( ).n n d f d d f− = (4.5.4) Khi tính vi phân cấp cao ta chú ý rằng dx là một số tuỳ ý và không phụ thuộc x ( dx x= Δ ), nên khi lấy vi phân theo x phải xem nó là hằng số. Trong trường hợp đó ta sẽ có 2 2 ( ) ( ) ( )d y d dy d y dx dy dx y dx dx y dx′ ′ ′′ ′′= = = = = . 3 2 2 3 ( ) ( )d y d d y d y dx y dx′′ ′′′= = = . Bằng cách quy nạp ta chứng minh được rằng ( )n n n d y y dx= . Do đó ( ) n n n d y y dx = . (4.5.5) Như vậy, ký hiệu trên có thể xem như một phân số. Nhờ công thức (4.5.5) ta dễ dàng biến đổi công thức Leibnitz thành công thức của vi phân. Nhân cả hai vế của (4.5.3) với dxn ta sẽ được 0 ( ) . n n k n k k n k d fg C d f d g− = = ∑ (4.5.6)
  • 20. 20 Chú ý trong công thức (4.5.6) ta sẽ xem 0 0 ,d f f d g g= = . Ví dụ 3: Cho y=f(x2 ) với f là hàm khả vi. Tính d2 y. Ta có: 2 2 ( )dy f x xdx′= , Lấy vi phân lần thứ hai ta được 2 2 2 2 2 2 2 2 2 2 2 2 ( ) 2 ( ) ( ( )) 2 ( ) ( ).2 2 2 ( ) ( ) . d y d f x x dx f x dx xd f x dx f x dx xf x xdx dx d y x f x f x dx ⎡ ⎤ ⎡ ⎤′ ′ ′= = +⎣ ⎦ ⎣ ⎦ ⎡ ⎤′ ′′= +⎣ ⎦ ⎡ ⎤′′ ′= +⎣ ⎦ Ví dụ 4: Xét hàm y = arctgx. Ta hãy tính y(n) theo y. Vì x = tgy nên 2 2 1 cos cos sin( ) 21 y y y y x π ′ = = = + + Lấy đạo hàm lần thứ hai theo x (và nhớ rằng y là hàm của x) ta được sin .sin( ) cos .cos( ) . 2 2 y y y y y y π π⎡ ⎤ ′′ ′= − + + +⎢ ⎥ ⎣ ⎦ 2 2 cos .cos(2 ) cos .sin(2 ) 2 2 2 cos .sin 2( ). 2 y y y y y y π π π π = + = + + = + Lấy đạo hàm lần nữa ta được (3) 2 3 3 2sin cos .sin 2( ) 2cos cos2( ) . 2 2 2cos cos(3 2. ) 2cos sin 3( ) 2 2 y y y y y y y y y y y π π π π ⎡ ⎤ ′= − + + +⎢ ⎥ ⎣ ⎦ = + = + Một cách tổng quát ( ) ( 1)!cos .sin ( ) 2 n n y n y n y π = − + . 4.6 Công thức Taylor Trước đây, ta đã biết nếu hàm f(x) khả vi tại điểm 0x U∈ , trong đó U là tập mở trong , thì ta có thể biểu diễn số gia của hàm số dưới dạng 0 0 0( ) ( ) ( ) ( )f x h f x f x h hο′+ − = + trong đó ( )hο là vô cùng bé bậc cao hơn so với h. Công thức này cho biết cách tính giá trị của f(x) trong lân cận của điểm x0 khi biết giá trị f(x0) và đạo hàm 0( )f x′ . Vấn đề đặt ra là nếu biết thêm các đạo hàm cấp cao của hàm f(x) tại x0, ta có thể biết chính xác hơn giá trị của hàm f(x) trong lân cận x0 hay không? 4.8.1 Công thức Taylor
  • 21. 21 21 Định lí 4.6.1 Cho [ ]: ,f a b → . Nếu hàm f(x) khả vi (n+1) lần trong khoảng (a,b), thì với bất kì điểm ( , ), ( , )c a b x a b∈ ∀ ∈ mà x c≠ ta luôn có 2 ( ) ( 1) 1 ( ) ( ) ( ) ( ) ( ) ( ) ... 1! 2! ( ) ( ) ( ) ( ) ! ( 1)! n n n n f c f c f x f c x c x c f c f c x c x c n n + + ′ ′′ = + − + − + + + − + − + (4.6.1) trong đó c là một số nằm giữa x và c. Chứng minh: Trước hết ta hãy tìm đa thức ( )nP x sao cho ( ) ( ) ( ) ( ); ( ) ( );...; ( ) ( )n n n n nP c f c P c f c P c f c′ ′= = = . (6.4.2) Thật vậy đa thức ( )nP x phải tìm được viết dưới dạng 2 0 1 2( ) ( ) ( ) ... ( )n n nP x a a x c a x c a x c= + − + − + + − (4.6.3) khi đó 0( )nP c a= và 1 1 2( ) 1. 2 ( ) ... . ( )n n nP x a a x c n a x c −′ = + − + + − 1 2 2 3 2 ( 1) 1 ( ) ( ) 2.1. 3.2. ( ) ... ( 1) ( ) ( ) 2! ................. ( ) ( 1)( 2)...2.1. ( 1)...3.2.1. ( ) n n n n n n n n n P c a P x a a x c n n a x c P c a P x n n a n n a x c − − − ′ = ′′ = + − + + − − ′′ = = − − + − − ( ) ( ) !n n nP c n a= Thay các hệ số a0, a1, …, an vào (4.6.3), đa thức P(x) phải tìm có dạng ( ) 2( ) ( ) ( ) ( ) ( ) ( ) ( ) ... ( ) 1! 2! ! n n n f c f c f c P x f c x c x c x c n ′ ′′ = + − + − + + − (4.6.4) Bây giờ ta đặt ( ) ( ) ( )n nR x f x P x= − (4.6.5) Theo giả thiết ( ) ( ) ( ) ( ) ... ( ) 0n n n n nR c R c R c R c′ ′′= = = = = (4.6.6) Mặt khác, nếu đặt 1 ( ) ( )n G x x c + = − (4.6.7) thì cũng có ( ) ( ) ( ) ( ) ... ( ) 0n G c G c G c G c′ ′′= = = = = và ( 1) ( ) ( 1)!n G x n+ = + Giả sử ( , ), ,x a b x c∈ ≠ từ (4.6.6) và (4.6.7) ta có ( ) ( ) ( ) ( ) ( ) ( ) n n nR x R x R c G x G x G c − = −
  • 22. 22 Áp dụng định lí Cauchy vào tỉ số trên ta được 1 1 ( ) ( ) ( ) ( ) n nR x R c G x G c ′ = ′ với c1 nằm giữa x và c Cũng từ các hệ thực (4.6.6) và (4.6.7) ta có 1 1 2 1 1 2 ( ) ( ) ( ) ( ) ( ) ( ) ( ) ( ) n n n nR c R c R c R c G c G c G c G c ′ ′ ′ ′′− = = ′ ′ ′ ′′− với c2 nằm giữa c1 và c Như vậy, áp dụng (n+1) lần định lí Cauchy ta được ( 1) ( 1) ( ) ( ) ( ) ( ) n n n n R x R c G x G c + + = Theo định nghĩa của hàm G(x) ta có 1 ( ) ( 1)!,n G x n+ = + do đó 1 ( ) ( 1).n G c n+ = + Từ đây suy ra ( 1) 1( ) ( ) ( ) ( 1)! n nn n R c R x x c n + + = − + . (4.6.9) Từ (4.6.5) 1 1 ( 1) ( 1) ( ) ( ) ( ) ( ).n n n n n nR x f x P x f x+ + + + = − = Từ đây ta nhận được ( 1) ( 1)( ) ( ) ( ) ( 1)! n n n f c R x x c n + + = − + . (4.6.10) Cuối cùng 2 ( ) ( 1) 1 ( ) ( ) ( ) ( ) ( ) ( ) ( ) ( ) ... 1! 2! ( ) ( ) ( ) ( ) ! ( 1)! n n n n n n f x P x R x f c f c f c x c x c f c f c x c x c n n + + = + ′ ′′ = + − + − + + + − + − + trong đó c là một số nằm giữa x và c, định lí được chứng minh. Người ta thường gọi công thức (4.6.1) là công thức Taylor và biểu diễn một hàm số f(x) dưới dạng (4.6.1) là khai triển Taylor của hàm số f(x) tại điểm x = c. 4.8.2 Khai triển Maclaurin Ta thấy khi c = 0, thì (4.6.1) có dạng
  • 23. 23 23 ( ) ( 1) 2 1(0) (0) (0) ( ) ( ) (0) ... 1! 2! ! ( 1)! n n n nf f f f c f x f x x x x n n + +′ ′′ = + + + + + + (4.6.11) trong đó c nằm giữa x và 0. Đặt c x= θ , trong đó 0 1θ< < , khi đó (4.6.11) trở thành ( ) ( 1) 2 1(0) (0) (0) ( ) ( ) (0) ... 1! 2! ! ( 1)! n n n nf f f f x f x f x x x x n n θ+ +′ ′′ = + + + + + + (4.6.12) Công thức (4.6.12) gọi là khai triển Maclaurin của hàm f, trong đó 1 1( ) R ( ) . ( 1)! n n n f x x x n θ+ + = + Chú ý: a) Trong định lý Taylor ta thường viết x = c + h, khi đó công thức (4.6.1) có dạng 2 ( ) ( 1) 1 ( ) ( ) ( ) ( ) ... 1! 2! ( ) ( ) . ! ( 1)! n n n n f c f c f c h f c h h f c f c h h h n n θ+ + ′ ′′ + = + + + + + + + + (4.6.13) b) Bây giờ trong công thức trên, nếu thay h bởi dx và nhớ rằng 2 2 ( ) ( ) ( ), ( ) ( ),..., ( ) ( )n n n f c dx df c f c dx d f c f c dx d f c′ ′′= = = và ( 1) 1 1 ( ) ( )n n n f dx d fξ ξ+ + + = , ta có thể biểu diễn khai triển trên dưới dạng 2 11 1 1 ( ) ( ) ( ) ... ( ) ( ) 2! ! ( 1)! n n f c df c d f c d f c d f n n ξ+ Δ = + + + + + (4.6.11) trong đó . , 0 1c hξ θ θ= + < < . Ví dụ 1: Trước hết hãy xét f(x) = ex với x∈ Khi đó ta có: ( ) * ( ) 0 ( ) * ( ) . (0) 1, ( ) . k x k k x f x e k f e f x e kθ θ = ∀ ∈ = = = ∀ ∈ Theo công thức (4.6.12) 2 1 1 ... 1! 2! ! ( 1)! n n x xx x x x e e n n θ + = + + + + + + với x∈ và 0 1θ< < hay 2 1 ... ( ) 1! 2! ! n x nx x x e x n ο= + + + + + với x∈ . (4.6.15) Ví dụ 2: Xét hàm số f(x) = sin x với x∈ . Ta thấy ( ) ( ) sin( ) 2 k k f x x π = + với k ∈ , do đó
  • 24. 24 (2 ) (0) 0, (0) sin 0n f f nπ= = = (2 1) 1 11 ( 1) (0) sin( ) ( 1) ( 1,2,3,...) 2 | ( )| ( ) . ( 1)! ( 1)! m m nn n n f m m xx R x f x n n π π θ − − ++ + = − = − = = ≤ + + Vì vậy bằng cách đặt trong công thức (4.6.12) n = 2m, ta được 3 5 2 1 1 2sin ... ( 1) 1! 3! 5! (2 1)! m m m x x x x x R m − − = − + + + − + − (4.6.16) trong đó 2 2 ( ) (2 )! m m x R x m ≤ hay 3 2 1 1 2 sin ... ( 1) ( ) 3! (2 1)! n n nx x x x x n − − = − + + − + ο − . Ví dụ 3: Tương tự ta có 2 2 2 1 cos 1 ... ( 1) ( ) 2! (2 )! n n nx x x x n ο + = − + + − + . (4.6.17) trong đó 2 1 2 1 ( ) cos (2 1)! n n x x x n θ + + ο = + với 0 1θ< < . Ví dụ 4: Xét khai triển của hàm 1 ( ) 1 f x x = + với 1x ≠ − Ta thấy ( ) 1 1 ! ( 1) 1 (1 ) n n n n x x + ⎛ ⎞⎟⎜ = −⎟⎜ ⎟⎟⎜⎝ ⎠+ + , từ đó ta có 2 ( ) (0) 1, (0) 1, (0) ( 1) 2!,..., (0) ( 1) !n n f f f f n′ ′′= = − = − = − Sử dụng công thức (4.6.12) ta được: 21 1 ... ( 1) ( ) 1 n n n x x x x x ο= − + − + − + + (4.6.18) trong đó 1 1 1 1 ( ) ( 1) (1 ) n n n n x x xθ + + + ο = − + . Ví dụ 5: Khai triển f(x) = ln(1+x) với x>−1 Ta nói rằng 1 1 1 ! (0) 0, ( ) , ( ) ( 1) 1 (1 ) n n n n f f x f x x x + + ′= = = − + + Do đó
  • 25. 25 25 2 3 1 ln(1 ) ... ( 1) ( ) 2 3 n n nx x x x x x n − + = − + − + − + ο trong đó 1 1 1 1 ( ) ( 1) . 1 (1 ) n n n n x x n xθ + + ο = − + + . Ví dụ 6: Xét khai triển ( ) (1 ) , , 0f x x α α α= + ∈ ≠ . Tương tự như trên, ta có thể chứng minh được rằng 2( 1) (1 ) 1 ... 2! ( 1)...( 1) ( ). ! n n x x x n x x n α α α α α α α − + = + + + + − − + + + ο (4.6.20) 4.7 Qui tắc L’hospital để khử dạng vô định Trước hết ta xét giới hạn ( ) lim ( )x c f x g x→ trong trường hợp f(x) và g(x) dần tới 0 khi x c→ . Trường hợp đặc biệt của giới hạn này là: Nếu như f liên tục tại điểm x0, tức là 0 0 0 0lim( ( ) ( )) lim( ) 0 x x x x f x f x x x → → − = − = thì ta gọi giới hạn 0 0 0 ( ) ( ) lim x x f x f x x x→ − − , nếu tồn tại, là đạo hàm 0( )f x′ . Cho nên ta hy vọng rằng bằng cách sử dụng những định lý về đạo hàm ta có thể khử được một số dạng vô định của giới hạn. Các quy tắc sau đây gọi chung là quy tắc L’Hospital. 4.8.1 Dạng vô định 0 0 Hãy xét dạng vô định ( ) lim ( )x c f x g x→ , trong đó lim ( ) 0 lim ( ) x c x c f x g x → → = = , với c có thể vô hạn hoặc hữu hạn. Định lý 4.7.1 Giả sử i) f và g là các hàm liên tục trên [a,b] và ( , )c a b∈ sao cho f(c) = g(c) = 0 ii) Nếu trong một lân cận nào đó của điểm c (có thể trừ điểm c) tồn tại các đạo hàm ,f g′ ′ với ( ) 0g x′ ≠ . iii) Ngoài ra tồn tại ( ) lim ( )x c f x g x→ ′ ′ hữu hạn. Khi đó ( ) ( ) lim lim ( ) ( )x c x c f x f x g x g x→ → ′ = ′ . (4.7.1) Chứng minh: Theo định lí Cauchy
  • 26. 26 ( ) ( ) ( ) ( ) ( ) ( ) ( ) ( ) f x f x f c f g x g x g c g ξ ξ ′− = = ′− trong đó ξ nằm giữa c và x Khi x c→ thì cξ → , suy ra ( ) ( ) ( ) lim lim lim ( ) ( ) ( )x c c x c f x f f x g x g g xξ ξ ξ→ → → ′ ′ = = ′ ′ . Chú ý: Nếu cả hai đạo hàm f ′ và g′ vẫn tiến tới 0 khi x c→ và chúng là các hàm khả vi trong lân cận của c thì ta có thể áp dụng quy tắc L’Hospital một lần nữa, một cách tổng quát: ( ) ( ) ( ) ( ) ( ) lim lim ... lim ( ) ( ) ( ) n nx c x c x c f x f x f x g x g x g x→ → → ′′ = = = ′′ . Ví dụ 1: 0 0 lim lim 2 1ln( 1) 1 x x x x x x e e e e x x − − → → − + = = + + Ví dụ 2: 2 20 0 sin (cos 1) 1 lim lim arcsin 1 1x x x x x x x x x→ → − − − = = − − − 2 20 0 2 2 0 0 (cos 1) lim 1 lim 1 1 sin 1 lim 1 lim 1. x x x x x x x x x x x → → → → − = − = − − − − = − = − Chú ý: quy tắc được phát biểu trên vẫn đúng cho cả giới hạn phải, giới hạn trái mà việc chứng minh được làm tương tự. Ví dụ 3: Tìm giới hạn 2 2 cos lim ( ) 2 x x I x π π→ = − Trước hết ta thấy theo quy tắc L’Hospital 2 sin lim 2( ) 2 x x I x π π→ − = − . Mặt khác 2 2 sin sin lim , lim 2( ) 2( ) 2 2 x x x x x x π ππ π− + → → − − = +∞ = −∞ − − , suy ra giới hạn I không tồn tại. Quy tắc L’Hospital còn đúng cho trường hợp ( ) 0,f x → ( ) 0g x → khi x → +∞ hay .x → −∞ Ta xét trường hợp khi x → +∞ . Định lí 4.7.2 Giả sử f và g là những hàm xác định trên (a,+∞ ) sao cho: i) lim ( ) lim ( ) 0, x x f x g x →+∞ →+∞ = = ii) f và g khả vi trên (a,+∞ ) và ( ) 0g x′ ≠ khi x đủ lớn,
  • 27. 27 27 iii) ( ) lim ( )x f x A g x→+∞ ′ = ′ . Khi đó ( ) lim ( )x f x A g x→+∞ = . (4.7.2) Chứng minh: Đặt 1 x y = , ta thấy khi x → +∞ thì 0y + → Ta có 1 ( ) ( ) 1( ) ( ) f f x y g x g y = . Nếu đặt 1 1 1 1 ( ) ( ), ( ) ( )f y f g y g y y = = thì 1 1 ( )( ) . ( ) ( ) f yf x g x g y = Suy ra 1 1 0 0 1 1 ( ) ( )( ) lim lim lim ( ) ( ) ( )x y y f y f yf x g x g y g y+ +→+∞ → → ′ = = = ′ 2 0 0 2 1 1 ( )( ) ( ) lim lim 1 1 ( ) ( )( ) y y f y f xy A g x g y y + + → → ′ − ′ = = = ′ ′ − , Do đó ( ) lim ( )x f x A g x→+∞ = , điều phải chứng minh. 4.8.2 Dạng vô dịnh ∞ ∞ Định lí 4.7.3 Giả sử i) Điểm ( , )c a b∈ và trong lân cận nào đó của điểm c (trừ điểm c) các hàm f, g khả vi và ( ) 0g x′ ≠ . ii) lim| ( )| x c f x → = +∞ và lim| ( )| x c g x → = +∞ . iii) ( ) lim ( )x c f x A g x→ ′ = ′ . Khi đó ( ) lim ( )x c f x A g x→ = . (4.7.3) Chứng minh: Do ( , )c a b∈ và ( ) lim ( )x c f x A g x→ ′ = ′ nên với 0ε > tuỳ ý cho trước, ta có thể tìm được một số 0δ > sao cho ( ) ( ) f t A A g t ε ε ′ − < < + ′ khi t cδ δ− < − < hay khi c t cδ δ− < < +
  • 28. 28 Chọn 0x c δ= + thì ( ) ( ) f t A A g t ε ε ′ − < < + ′ khi 0c t x< < . (4.7.4) Mặt khác, với ( , )x a b∈ sao cho 0c x x< < , khi áp dụng định lí Cauchy ta có 0 0 ( ) ( ) ( ) ( ) ( ) ( ) f x f x f g x g x g ξ ξ ′− = ′− với 0x xξ< < . (4.7.5) Hơn nữa 0 0 00 ( ) 1 ( ) ( ) ( ) ( ) . ( )( ) ( ) ( ) 1 ( ) f x f x f x f x f x g xg x g x g x g x − − = − − , suy ra 0 0 00 ( ) 1 ( ) ( )( ) ( ) . ( )( ) ( ) ( ) 1 ( ) g x f x f xf x g x f xg x g x g x f x − − = − − . (4.7.6) Từ (4.7.5) và (4.7.6) ta thấy 0 0 ( ) 1 ( ) ( ) ( ) . ( )( ) ( ) 1 ( ) g x f x f g x f xg x g f x ξ ξ − ′ = ′ − . (4.7.7) Vì 0c x xξ< < < nên ( ) ( f A A g ξ ε ε ξ ′ − < < + ′ (4.7.8) Bây giờ cho x c→ , theo giả thiết lim| ( )| , x c f x → = +∞ lim| ( )| x c g x → = +∞ suy ra 0 0 ( ) 1 ( ) lim 1, ( ) 1 ( ) x c g x g x f x f x → − = − (4.7.9) nên ta có thể viết 0 0 ( ) 1 ( ) 1 ( ), ( ) 1 ( ) g x g x x f x f x δ − = + − (4.7.10) trong đó ( ) 0xδ → khi x c→ Theo hệ thức (4.7.7), ta có
  • 29. 29 29 ( ) ( )(1 ( )) ( )(1 ( )) ( ) f x A x A x g x ε δ ε δ− + < < + + . (4.7.11) Do ε nhỏ tuỳ ý, nên ( ) lim ( )x c f x A g x→ = . Định lí được chứng minh. Ví dụ 4: 2 20 0 0 ln 1 ln lim lim lim cot1 cot 1x x x x x gxx gx x + + + → → → = = + + 2 2 20 0 0 0 2 1 1 1 sin lim . lim lim . lim 0. 11 1 sin x x x x xx xx x x + + + + → → → → = = = + +− Chú ý: Quy tắc L’Hospital có thể dùng để khử các dạng vô định khác 0 ;0. ; ;1∞ ∞ − ∞ ∞ ∞ . Ví dụ 5: Tính 0 lim ln x I x xα + → = do 0α > . 1 0 0 0 1 ln 1 lim lim lim 0 x x x x xI x x x α α α αα+ + +− − − → → → = = = − = − do 0α > . Ví dụ 6: Tính 2 20 1 1 lim( ) sinx I x x→ = − 2 2 2 2 2 20 0 sin 2sin cos 2 lim lim sin 2 sin 2 sin cosx x x x x x x x x x x x x x→ → − − = = + 2 20 sin 2 2 lim 2 sin sin 2x x x x x x x→ − = + 2 20 2 2 2 2 os2 2 lim 2sin 4 sin 2 2 cos2 2sin lim sin 2 sin 2 cos2 x c x I x x x x x x x x x x x → − = + + = − + + 20 2 2 2 1 lim 3sin 2 cos2 1 2 sin sin x x x x x x x → = − = − + + . Ví dụ 7: ln 0 0 0 ) lim lim 1,x x x x x a x e e+ + → → = = = 1 ln 1 1 1 1 ) lim lim . x x x x x b x e− − → → = Ta thấy 1 1 1 ln lim lim 1 1 1x x x x x→ → = = − , nên 1 1 1 lim x x x e− → = .
  • 30. 30 Cuối cùng ta hãy xét độ tăng của các hàm , vµ logx m aa x x Ví dụ 8: Cho a>1 và nếu m là số tuỳ ý, thì lim x mx a x→+∞ = +∞ (4.7.12) Do 2 1 2 1ln ln ln ln 1 ... 1! 2! ! ( 1)! n x n x n na a a a a a x x x x n n θ + + = + + + + + + . Bây giờ ta hãy chọn số tự nhiên n sao cho n>m. Với x>0, từ (4.7.13) ta thấy ln , ! n x na a x n > hay ln ! x n n m m a a x nx − > . Từ đây suy ra lim x mx a x→+∞ = +∞ . Ví dụ 9: cho a>1 và m>0, log lim 0a mx x x→+∞ = (4.7.14) Thật vậy, đặt log .log ,m y m a ax a y x m x= ⇒ = = khi x → +∞ thì y → +∞ . Do đó log 1 ,a m y x y mx a = theo ví dụ trên log 1 lim lim 0a m yx y x y mx a→+∞ →+∞ = = , điều phải chứng minh. Từ các ví dụ trên ta thấy khi x → +∞ , hàm ax với a>1 tăng nhanh hơn bất cứ hàm luỹ thừa nào của x. Khi x → +∞ , hàm log , 1a x a > tăng chậm hơn bất kỳ hàm luỹ thừa xm với số mũ dương. 4.8 Khảo sát hàm số 4.8.1 Khảo sát đường cong cho dưới dạng phương trình hiện Xét hàm số ( ), ( , )y f x x a b= ∈ . (4.8.1) Ở trường phổ thông, để khảo sát sự biến thiên của hàm số ta thường tìm cực đại, cực tiểu của hàm số theo qui tắc I và quy tắc II và tìm điểm uốn của đồ thị. Ví dụ 1: Ta hãy xét hàm số 2 ( ) 1 x f x x = + . Miền xác định của hàm số là ( , )Df = −∞ +∞ . Ta có 2 2 2 2 2 3 1 2 ( 3) ( ) , ( ) ( 1) ( 1) x x x f x f x x x − − ′ ′′= = + + . Theo dấu của đạo hàm f’(x) ta thấy rằng hàm số tăng trong khoảng (−1,1), giảm trong các khoảng ( ,1−∞ ) và (1,+∞ ). Do đó tại điểm –1 hàm số đạt cực tiểu, tại điểm 1 hàm số đạt cực đại:
  • 31. 31 31 1 1 ( 1) , (1) 2 2 f f− = − = . Đạo hàm cấp hai âm trong khoảng ( , 3−∞ ) và trong khoảng (0, 3 ), dương trong khoảng (– 3 ,0) và ( 3 ,+∞). Do đó những điểm (− 3 3, 4 − ); (0,0); ( 3 , 3 4 ) là những điểm uốn của đồ thị. Ngoài ra ta chú ý rằng 2 2 lim 0 vµ lim 0 1 1x x x x x x→+∞ →−∞ = = + + . Sau đây chúng ta sẽ giới thiệu hai định lí khi f(x) có đạo hàm cấp cao mà phần chứng minh của nó độc giả có thể đọc trong cuốn [1]. Định lí 4.8.1 Giả sử trong lân cận của điểm x = c hàm f(x) có đạo hàm cấp n và đạo hàm cấp n tại điểm x = c liên tục. Ngoài ra giả sử ( ) ( ) ( ) 0 ví i 1 vµ ( ) 0.k n f c k n f c= ≤ < ≠ Khi đó i) Nếu n chẵn, ( ) ( ) 0n f c > , thì hàm f(x) đạt cực tiểu địa phương tại x = c ii) Nếu n chẵn, ( ) ( ) 0n f c < , thì hàm f(x) đạt cực đại địa phương tại x = c iii) Nếu n lẻ thì hàm f(x) không đạt cực trị tại x = c. Định lí 4.8.2 Giả sử trong lân cận điểm x = d hàm f(x) có đạo hàm cấp m và đạo hàm cấp m tại điểm x = d liên tục. Ngoài ra giả sử ( ) ( ) ( ) 0 ví i 1 , nh- ng ( ) 0k m f d k m f d= ≤ < ≠ . Khi đó nếu m lẻ thì (d, f(d)) là điểm uốn của đồ thị. Kết hợp hai định lí trên ta có điều cần nhớ sau đây: Nếu 0 0( ) 0, ( ) 0f x f x′ ′′= ≠ thì hàm số đạt cực trị tại điểm x = x0 (đạt cực tiểu nếu 0( )f x′′ >0, cực đại nếu 0( )f x′′ <0 Nếu 0( )f x′′ = 0, (3) 0( ) 0f x ≠ thì điểm (x0, f(x0)) là điểm uốn. Ví dụ 2 Hãy tìm cực trị địa phương và điểm uốn của hàm số 7 6 5 41 1 1 1 ( ) 7 6 5 4 f x x x x x= + − − . Ta có 6 5 4 3 3 3 2 3 2 2 2 2 1 2 1 1 1 1 6 3 6 1 ( ) ( ) ( ) ( ) ( ) ( )( ) ( )( )( ), f x x x x x x x x x x x x f x x x x x x x x xα α ′ = + − − = + − − = + − ′′ = + − − = + − − trong đó 1 2 1 1 1 73 1 73 12 12 ( ), ( ).α α= + = − Do 1 2 5 10 8 7 12 12 12 12 ;α α< < − < < − , cho nên 2 11 0 1.α α− < < < < Ngoài ra (3) 4 3 2 ( ) 30 20 12 6 .f x x x x x= + − −
  • 32. 32 Phương trình ( ) 0′ =f x có nghiệm x= 0, –1, 1 và phương trình ( ) 0′′ =f x có nghiệm x = 0, –1, 1 2,α α . Ta thấy 3 4 0 0 0 0 0 0( ) ( ) ( ) ( ) , ( ) ;f f f f′ ′′= = = < 3 1 0 1 0( ) ( ) , ( ) ;f f′′ − = − > 1 0 1 0( ) , ( ) .f f′ ′′= > Cho nên hàm số đạt cực đại tại x = 0, cực tiểu tại x = 1, điểm uốn tại x = –1, 1 2, .α α Chú ý đối với hàm số này ta có thể tìm cực đại, cực tiểu theo qui tắc I và xét dấu đạo hàm cấp hai để tìm điểm uốn. 4.8.2 Đường cong cho dưới dạng tham số Cho hệ hai phương trình ( ) , ( , ) ( ) x x t t y y t α β =⎧ ∈⎨ =⎩ . (4.8.2) Khi đó với mỗi giá trị ( , )α β∈t hệ phương trình (4.8.2) cho ta một điểm M(x,y) tương ứng trong mặt phẳng Oxy và khi t biến thiên trong ( , )α β điểm M vạch nên một đường cong Γ nào đó trong mặt phẳng, vì thế ta gọi hệ phương trình (4.8.2) là hệ phương trình tham số của đường cong Γ , trong đó t là tham số. Ví dụ 3: Phương trình tham số của đường thẳng đi qua hai điểm A(a,c) và B(b,d) là , x mt a t y nt b = +⎧ ∈⎨ = +⎩ (4.8.3) trong đó m = b − a, n = d − c. Ví dụ 4: Phương trình tham số của ellip 2 2 2 2 1 x y a b + = là 0 2 cos ví i [ , ] sin x a t t y b t π =⎧ ∈⎨ =⎩ . (4.8.4) Để khảo sát đường cong cho dưới dạng tham số ta cần thực hiện các bước sau đây: a) Tìm tập xác định của các hàm số x = x(t), y= y(t) b) Xét chiều biến thiên của x, y theo t c) Tìm các đường tiệm cận: i ) Nếu 0 ( ) lim t t t y → →±∞ = ±∞ và 0 ( ) lim t t t x a → →±∞ = (hữu hạn) thì đường cong có tiệm cận đứng là x = a. ii) Nếu 0 ( ) lim t t t x → →±∞ = ±∞ và 0 ( ) lim t t t y b → →±∞ = (hữu hạn) thì đường cong đó có tiệm cận ngang là y = b.
  • 33. 33 33 iii) Nếu 0 ( ) lim t t t x → →±∞ = ∞ , 0 ( ) lim t t t y → →±∞ = ∞ , và nếu 0 0 ( ) ( ) lim , lim ( ) t t t t t t y a b y ax x→ → →±∞ →±∞ = = − thì đường cong có tiệm xiên là y = ax+b. Ví dụ 5: Khảo sát và vẽ đường cong axtrôit 2 2 2 3 3 3 0,x y a a+ = > . Dễ thấy phương trình tham số của đường cong nói trên là 3 3 0 cos , ( , ), sin x a t t a y a t ⎧ =⎪ ∈ −∞ +∞ >⎨ =⎪⎩ . (4.8.5) Trước hết, ta thấy đường cong y không có tiệm cận. Hơn nữa x,y là các hàm tuần hoàn với chu kỳ 2π nên ta chỉ cần khảo sát đường cong đã cho trong đoạn [0,2 ]π . 2 2 3 3 0 0 2 2 2 3 3 0 0 2 2 2 ( ) cos .sin khi ; ; ; ; ( ) sin .cos khi ; ; ; ; . x t a t t t y t a t t t π π π π π π π π ′ = − = = ′ = = = Cuối cùng ta nhận xét rằng đối với đường cong axtrôit ta có 2 2 3 tg 3 sin cos cos sin t t ydy a t t t dx x a t t ′ = = = − ′ − . Do đó 0 dy dx = tại 0 2; ;t π π= và tại các điểm này tiếp tuyến thẳng đứng (xem hình 4.8.1).
  • 34. 34 Hình 4.8.1 Ví dụ 6: Khảo sát và vẽ đường cong cho bởi phương trình 3 3 3 0 0,x y axy a+ − = > . (4.8.6) Ta thấy rằng khi thay x bởi y và y bởi x thì phương trình (4.8.6) không thay đổi. Do đó, đồ thị của nó đối xứng nhau qua đường phân giác thứ nhất. Ta đặt y = tx và thay vào phương trình trên ta được 3 3 3 2 3 0x x t ax t+ − = , từ đây suy ra 2 3 3 3 3 1 1 , at at x y t t = = + + với 1t ≠ − . (4.8.7) Lấy vi phân ta thu được 3 3 2 3 1 2 1 ( ) 3 ; ( ) 0 khi vµ (1 ) 2 t x t a x t t t − ′ ′= = = +
  • 35. 35 35 3 3 3 2 2 ( ) 3 ; ( ) 0 khi 2 (1 ) t y t at y t t t − ′ ′= = = + . Khi 1t → − thì x, y đều dẫn tới ∞ và 1 1 lim lim 1 t t y a t x→− →− = = = − 3 3 31 1 3 3 lim( ) lim[ ] 1 1t t at at b y ax t t→− →− = − = + = + + 2 3 21 1 1 ( 1) 3 lim 3 lim 1 ( 1)( 1)t t t t t a a a t t t t→− →− + + = = = − + + − + . Vậy tiệm cận xiên của đường cong là y = − x − a. Từ các kết quả trên ta có bảng biến thiên. Cuối cùng ta hãy chú ý thêm rằng 3 3 (2 ) 1 2 dy t t dx t − = − Do đó 3 3 1 0 khi 0, 2, khi , 2 dy dy t t t t dx dx = = = = ∞ = ∞ = . Các tiếp tuyến của đường cong ứng với hai giá trị = 0,t = 3 2t song song với trục Ox. Các tiếp tuyến ứng với hai giá trị = ∞ = 3 1 vµ 2 t t song song với trục Oy (xem hình 4.8.2)
  • 36. 36 3 3 3 0x y axy+ − = y x a= − x Hình 4.8.2 Hình 4.8.3 4.8.3 Khảo sát đường cong trong tọa độ cực a) Hệ tọa độ cực Trong mặt phẳng chọn một điểm O cố định và tia Ox đi qua điểm O. Ta gọi điểm O là cực, tia Ox gọi là trục cực. Hệ tọa độ xác định bởi cực và trục cực gọi là hệ tọa độ cực. Gọi OP là véc tơ đơn vị nằm trên tia Ox. Vị trí của điểm M trong mặt phẳng được xác định bởi véc tơ OM , nghĩa là xác định bởi góc ϕ = ( , )OP OM và =| |r OM , ϕ được gọi là góc cực, r được gọi là bán kính cực. Góc gọi là góc định hướng, lấy giá trị dương nếu chiều quay OP đến trùng với OM ngược chiều kim đồng hồ và lấy giá trị âm nếu ngược lại. Cặp số có thứ tự ϕ( , )r gọi là tọa độ của điểm M trong mặt phẳng. b) Mối liên hệ giữa tọa độ Descartes vuông góc và tọa độ cực Bây giờ ta lấy trục hoành trùng với trục cực và trục tung ứng với tia π ϕ = 2 ta được hệ tọa độ Descartes vuông góc. Gọi (x,y) và ϕ( , )r lần lượt là tọa độ của điểm M nói trên trong hệ tọa độ Descartes vuông góc và hệ tọa độ cực. Khi đó, ta có cos ví i 0 2 , 0 sin x r r y r ϕ ϕ π ϕ =⎧ ≤ < ≥⎨ =⎩ Cho hàm số ( )r f ϕ= . Trước khi khảo sát hàm số ta có nhận xét sau. Giả sử cho điểm M(x,y) nằm trên đồ thị. Gọi β là góc dương giữa véctơ OM và véctơ chỉ phương của tiếp tuyến với đồ thị tại điểm M. Gọi là góc dương giữa trục cực và tiếp tuyến, ta có. α ϕ β α ϕ β α ϕ − = − = + tg tg , tg 1 tg tg . (4.8.8) Mặt khác theo ý nghĩa hình học của đạo hàm
  • 37. 37 37 'sin cos tg 'cos sin dy r r dx r r ϕ ϕ α ϕ ϕ + = = − , trong đó ' dr r dϕ = (4.8.9) Thay vào (4.8.9) vào (4.8.8) ta được Hình 4.8.4 'sin cos sin 'cos sin cos tg 'sin cos sin 1 'cos sin cos r r r r r r r r ϕ ϕ ϕ ϕ ϕ ϕ β ϕ ϕ ϕ ϕ ϕ ϕ + − − = + + − Sau các phép biến đổi đơn giản tg ' r r β = . (4.8.10) Ví dụ 7: Hãy vẽ đường xoắn ốc lôgarit có phương trình: , 0, 0b r ae a bϕ = > > . (4.8.11) Ta thấy hàm số r xác định với mọi ϕ . Khi ϕ tăng r cũng tăng, khi 0ϕ = thì r = a, khi ϕ → +∞ thì r → +∞ , khi ϕ → −∞ thì 0r → và khi đó đường cong quấn vô hạn quanh cực O; O được gọi là điểm tiệm cận của đường cong. Theo công thức (4.8.9) ta có 1 tg ' r r a β = = , Do đó véc tơ chỉ phương của tiếp tuyến với đường cong luôn luôn tạo với OM một góc không đổi (xem hình 4.8.5). 0 β ϕ α H×nh 4.8.4
  • 38. 38 β Hình 4.8.5 Ví dụ 8: Hãy vẽ đường hoa hồng ba cánh có phương trình sin 3 , 0r a aϕ= > . Ở đây r là một hàm tuần hoàn với chu kì 2 3 π vì thế chỉ cần khảo sát hàm số trong một đoạn có độ dài bằng chu kì, chẳng hạn đoạn [0, 2 3 π ]. Ta có ' 3 cos3 ; ' 0 khi , , 6 2 r a r π π ϕ ϕ ϕ= = = = 1 tg tg3 ' 3 r r β ϕ= = . Đồ thị ứng với khoảng [0, 2 3 π ] gồm hai cánh, sau đo cho đồ thị quay các góc quanh cực ta sẽ có toàn bộ đồ thị (xem hình 4.8.6) Hình 4.8.6
  • 39. 39 39 4.9 Bài tập chương 4 4.1 Cho hàm 1 ( ) 1 x x f x e = + với 0x ≠ và f(0)=0. Chứng minh rằng hàm f(x) liên tục tại mọi điểm, nhưng (0) 0,f+ ′ = (0) 1f− ′ = . 4.2 Chứng minh rằng hàm 1 ( ) sinf x x x = với 0x ≠ , và f(0)=0 liên tục tại x = 0, nhưng không có đạo hàm bên trái và bên phải tại x = 0. 4.3 Chứng minh rằng hàm 2 nÕu lµ h÷u tØ ( ) 0 nÕu v« tØ x x f x x ⎧⎪ = ⎨ ⎪⎩ chỉ có đạo hàm tại x=0. 4.4 Dựa vào định nghĩa hãy tính f’(a) nếu ( ) ( ) ( ),f x x a xϕ= − trong đó ( )xϕ liên tục tại x=a. 4.5 Cho hàm y=sgnx được định nghĩa sau 1 nÕu 0, sgn 0 nÕu 0, 1 nÕu 0. x x x x − <⎧ ⎪ = =⎨ ⎪ >⎩ Chứng minh rằng |x|=xsgnx 4.6 Tính đạo hàm của các hàm số sau 1) y=|x| 2) y=x|x| 3) y=ln|x| với 0x ≠ . 4.7 Tính đạo hàm các hàm sau 1) y x x x= + + 2) 2 3 | ( 1) ( 1) |y x x= − + . 4.8 Tính y’ nếu 1) y=f(x2 ) 2) y=f(sin2 x)+ f(cos2 x) 3) y=f(f(f(x))), trong đó f(x) là hàm khả vi.
  • 40. 40 4.9 Chứng minh rằng hàm 2 1 sin nÕu 0 ( ) 0 nÕu 0 x x f x x x ⎧ ≠⎪ = ⎨ ⎪ =⎩ có đạo hàm gián đoạn. 4.10 Tính đạo hàm các hàm sau 1) y=xx 2) y=xslnx , trong đó s là hằng số. 4.11 Tính đạo hàm y’x nếu hàm số y được cho dưới dạng 2 2 2 2 cos , sint t x e t y e t= = . 4.12 Xác định miền tồn tại hàm ngược x = x(y) và tìm đạo hàm của nó nếu 1) y=x+lnx (x>0) 2) y=x+ex 4.13 Đưa về dạng F(x,y) = 0 (hay y = f(x)) phương trình các đường cong cho dưới dạng tham số. 1) x = acost, y = bsint 2) x = acos3 t, y = asin3 t 3) , 2 2 t t t t e e e e x y − − + − = = 4) x = tgt, y = cos2 t. 4.14 Cho hàm f(x)=x3 −2x+1. Hãy xác định (1)fΔ và (1)df nếu 0,1xΔ = . 4.15 Tìm vi phân của hàm 1) ( arctg ) a x d x a + , a là hằng số 2) (1 cos )d u− 3) ( )bt d bt e− − , b là hằng số. 4.16 Hãy tính 1) 3 6 9 2 ( 2 ) ( ) d x x x d x − − 2) 2 sin ( ) ( ) d x xd x 3) (tg ) (cot g ) d x d x 4.17 Hãy tìm hàm ( , )x xθ θ= Δ sao cho ( ) ( )f x x f x+ Δ − = = '( ),xf x xθΔ + Δ (0 1)θ< < nếu 1) 2 ( ) ( 0)f x ax bx c a= + + ≠ 2) ( ) x f x e= .
  • 41. 41 41 4.18 Cho hàm (1) ( , )f C∈ −∞ +∞ và hệ thức ( ) ( ) . '( )f x h f x h f x+ − = được nghiệm đúng ,x h R∀ ∈ . Chứng minh rằng ( )f x ax b= + trong đó a,b là hằng số. 4.19 Cho hàm số 3 ( ) , [ 1,1]f x x x= ∈ − . Hỏi có thể tìm được hai số 1 2, ( 1,1)x x ∈ − sao cho 2 1 2 1 ( ) ( ) '(0) f x f x f x x − = − hay không? 4.20 Cho hàm f(x) khả vi trên đoạn 1 2[ , ]x x , trong đó 1 2. 0x x > . Chứng minh rằng 1 2 1 21 2 1 ( ) ( ) ( ) ( ) x x f f f x f xx x ξ ξ ξ′= − − , trong đó 1 2x xξ< < . 4.21 Cho hàm f(x) liên tục trên [a,b] và có đạo hàm tại mọi điểm ( , )x a b∈ . Chứng minh rằng bằng cách áp dụng định lí Rolle đối với hàm ( ) 1 ( ) ( ) 1 ( ) 1 x f x x b f b a f a φ = ta sẽ thu được định lí Lagrange 4.22 Cho :f → chứng minh rằng nếu ( 1) (0) '(0) (0) ... (0) 0n f f f f − ′′= = = = = thì ( ) ( ) ( ) ! n n f x f x nx θ = trong đó 0 1θ< < . 4.23 Tìm đạo hàm cấp 2 của các hàm sau 1) cosx y e x= 2) 3x y a x= 3) 2 siny x x= . 4.24 Tìm đạo hàm cấp 3 của các hàm sau 1) sinx y e x− = 2) 2 lny x x= 3) cosy x x= . 4.25 Cho ( ) x af x xe= . Tìm (4) ( ) ( ) ( ), ( ), (0)n n f x f x f 4.26 Cho ( ) 1 x f x x = + . Chứng minh rằng với 2n ≥ ( ) 1 1 1.3.5...(2 3) (0) ( 1) 2 n n n n f n− − − = − .
  • 42. 42 4.27 Cho 2 1 ( ) 1 f x x = − . Chứng minh rằng ( ) ! khi 2 (0) 0 khi 2 1. n n n m f n m =⎧ = ⎨ = −⎩ 4.28 Cho hàm số 2 ( ) , 0 x af x x e a − = ≠ . Chứng minh rằng ( ) 2 ( 1)( 1) (0) n n n n n f a − − − = . 4.29 1) Cho * ( ) ,n f x x n N= ∈ . Chứng minh rằng ( ) (1) (1) (1) (1) ... 2 1! 2! ! n nf f f f n ′ ′′ + + + + = 2) Cho ( sin ), (1 cos )x a t t y a t= − = − . Tính 2 2 d y dx 3) Cho = =2 3 , 3t t x e y . Tính 2 2 d y dx . 4.30 Cho đa thức Legendre: 2 ( )1 ( ) [( 1) ] ( 0,1,2...) 2 . ! m m m m P x x m m = − = Chứng minh rằng ( )mP x thỏa mãn phương trình 2 (1 ) ( ) 2 ( ) ( 1) ( ) 0.m m mx P x xP x m m P x′′ ′− − + + = 4.31 Cho đa thức Lague: ( ) ( ) ( )x m x m mL x e x e− = thỏa mãn phương trình ( ) (1 ) ( ) ( ) 0m m mxL x x L x mL x′′ ′+ − + = 4.32 Chứng minh công thức: 1 1 ( ln ) !(ln ) n n n n k d x x n x kdx = = + ∑ với x>0. 4.33 Khai triển hàm 2 2 ( ) x x f x e − = đến số hạng chứa x5 . 4.34 Khai triển hàm ( ) 1x x f x e = − đến số hạng chứa x4 . 4.35 Tìm 3 số hạng đầu tiên của khai triển Taylor trong lân cận của điểm x=0. 1) (sinx)2 2) cos x .
  • 43. 43 43 4.36 Cho n số 1 2, ,..., na a a . Xác định x sao cho hàm số. 2 1 ( ) ( ) n k k x a xϕ = = −∑ có giá trị bé nhất. 4.37 Tìm 3 số hạng của khai triển hàm ( )f x x= theo các lũy thừa nguyên dương của hiệu x−1. 4.38 Khai triển hàm f(x)=xx −1 theo các lũy thừa nguyên dương của nhị thức (x−1) đến số hạng chứa (x−1)3 . 4.39 Áp dụng qui tắc L’hospital để tìm các giới hạn sau: 1) 1 1 1 lim( ) ln 1x x x→ − − 2) 2 1 0 sin lim( )x x x x→ 3) 20 2 ln(1 2 ) lim 2x x x x→ − + 4) 2 20 1 2 lim 2 x x e x x→ − − . 4.40 Hãy tìm giới hạn sau, xét xem có thể áp dụng qui tắc L’hospital hay không 1) 2 0 1 sin lim sinx x x x→ 2) sin lim sinx x x x x→∞ − + 3) sin 1 sin cos lim ( sin cos ) xx x x x x x x e→∞ + + + . 4.41 Tìm các giới hạn sau: 1) 0 1 1 lim( ) 1xx x e→ − − 2) 0 ln(sin ) lim ln(sin )x ax bx→ 3) 2 2 40 lim sin x x e x x→ − 4) 0 lim(sin )tgx x x → 5) 1 2 1 0 lim(1 ) x e x x x − − → + 6) 2cos 2 lim( ) x x tgx π → . 4.42 Chứng minh rằng khi 0x → ta có 1) 3 arctg 3 x x x− ∼ 2) ~ lnx x a a b x b − 3) 2 2 1 2 ~ 2x e x x− − 4) 2 2 ln(1 2 ) ~ 2x x x− + . 4.43 Cho hàm 6 1 ( ) sing x x x = với 0; (0) 0x g≠ = và 6 ( ) 2 ( ).f x x g x= + Xét cực trị địa phương của các hàm g, f tại điểm x=0. 4.44 Khảo sát và vẽ đồ thị hàm số 2 2 ( 1) 1 x y x − = + .
  • 44. 44 4.45 Khảo sát và vẽ đồ thị hàm số ln . x y x = 4.46 Khảo sát và vẽ đồ thị hàm số 2 x y x e− = . 4.47 Khảo sát và vẽ đồ thị hàm số 2 2 1 1y x x= + + − . 4.48 Khảo sát và vẽ đồ thị hàm số 1 2cosr ϕ= + .